Site Loader

Содержание

Закон Джоуля-Ленца



При прохождении электрического тока через металлический проводник электроны сталкиваются то с нейтральными молекулами, то с молекулами, потерявшими электроны.
Движущийся электрон либо отщепляет от нейтральной молекулы новый электрон, теряя свою кинетическую энергию и образуя новый положительный ион, либо соединяется с молекулой, потерявшей электрон (с положительным ионом), образуя нейтральную молекулу.
При столкновении электронов с молекулами расходуется энергия, которая превращается в тепло.
Любое движение, при котором преодолевается сопротивление, требует эатраты определенной энергии.

Так, например, для перемещения какого -либо тела преодолевается сопротивление трения, и работа, затраченная на это, превращается в тепло.
Электрическое сопротивление проводника играет ту же роль, что и сопротивление трения.

Таким образом, для проведения тока через проводник источник тока затрачивает некоторую энергию, которая превращается в тепло.

Переход электрической энергии в тепловую отражает закон Ленца — Джоуля
или закон теплового действия тока.

Русский ученый Ленц и английский физик Джоуль одновременно и независимо один от другого установили, что

при прохождении электрического тока по проводнику количество теплоты, выделяемое в проводнике, прямо пропорционально квадрату тока, сопротивлению проводника и времени, в течение которого электрический ток протекал по проводнику.

Это положение называется законом Ленца — Джоуля.
Если обозначить количество теплоты, создаваемое током, буквой Q (Дж),  ток, протекающий по проводнику — I, сопротивление проводника — R

и время, в течение которого ток протекал по проводнику — t, то закону Ленца — Джоуля можно придать следующее выражение:
Q = I2Rt.
Так как I = U/R и R = U/I, то Q = (U2/R) t = UIt.

Значение мощности, при выделении определённого количества тепла

Скачать можно здесь


(Подробно и доходчиво в видеокурсе «В мир электричества — как в первый раз!»)

Количество теплоты, выделяемое проводником с током — урок. Физика, 8 класс.

Проходя по проводнику, ток может оказывать некоторые действия: тепловое, химическое и магнитное.

 

Тепловое действие тока обусловлено тем, что свободные электроны, двигаясь с большой скорость, взаимодействуют с ионами металлов, ионами солей в растворах кислот и щелочей. Ионы начинают усиленно колебаться, двигаться, вращаться, то есть их энергия тоже повышается. Проводник или электролит нагревается.

Например, спираль лампочки раскаляется до такой температуры, что начинает излучать свет.

 

 

Электрическая энергия превращается в тепловую энергию проводника; часть рассеивается, часть используется в бытовых целях (для нагревания).

 

Работа, которую совершает электрический ток, определяется количеством теплоты, выделяемой проводником: Q = A, где \(A\) — работа тока, \(Q\) — количество теплоты.

 

Работу тока рассчитывают по формуле: A = U⋅I⋅t. Тогда количество теплоты, исходя из закона сохранения энергии, также будет равно: Q = U⋅I⋅t.


Согласно закону Ома U = IR. Подставляя эту формулу в предыдущую, получим: Q = I2⋅R⋅t.

Количество теплоты, которое выделяется в проводнике с током, прямо пропорционально квадрату силы тока, сопротивлению проводника и времени протекания тока.

В процессе своих экспериментов получили такой же результат Джеймс Джоуль в Англии и Эмилий Христианович Ленц в России. В их честь закон имеет двойное название: закон Джоуля-Ленца.

 

 

Джоуль Джеймс Прескотт (\(1818—1889\)) — английский физик, член Лондонского королевского общества. Он внёс значительный вклад в исследование электромагнетизма и тепловых явлений, в создание физики низких температур, в обоснование закона сохранения и превращения энергии. Именем Джоуля назвали единицу измерения работы и энергии в системе СИ.

 

 

Эмилий Христианович Ленц (\(1804—1865\)) — российский физик и электротехник, академик Петербургской АН (\(1830\)), ректор Санкт-Петербургского университета (с \(1863\)) — один из основоположников электротехники. С его именем связано открытие закона, определяющего тепловые действия тока, и закона, определяющего направление индукционного тока.

 

Преобразование электрической энергии в тепловую широко используется в электрических печах и различных электронагревательных приборах.

 

Состояние сети, когда по проводам и приборам проходит ток больше допустимого значения, называется перегрузкой. Опасность этого явления в тепловом действии тока, ведь при большой перегрузке изоляция проводников легко воспламеняется. Перегрузка может возникнуть при подключении устройств большой мощности через удлинитель (смотри рисунок и никогда так не делай!).

 

 

Для примера, перегрузка проводов на \(25\)% приводит к сокращению срока их службы где-то с \(20\) лет до \(3—5\) месяцев, а перегрузка проводов на \(50\)% — до нескольких часов.

Урок 30. закон джоуля-ленца. эдс — Физика — 10 класс

Физика, 10 класс

Урок 30. Закон Джоуля — Ленца. ЭДС

Перечень вопросов, рассматриваемых на уроке:

1) Работа электрического тока;

2) Мощность электрического тока;

3) Закон Джоуля — Ленца;

4) Сторонние силы;

5) Электродвижущая сила.

Глоссарий по теме

Работа тока на участке цепи равна произведению силы тока, напряжения на этом участке и времени, в течении которого совершалась работа.

Мощность тока равна отношению работы тока ко времени прохождения тока.

Количество теплоты, выделяемое проводником с током, равно произведению квадрата силы тока, сопротивления проводника и времени прохождения тока.

Любые силы, действующие на электрически заряженные частицы, за исключением электростатических (кулоновских) сил, называются

сторонними силами.

Электродвижущая сила (ЭДС) в замкнутом проводящем контуре равна отношению работы сторонних сил по перемещению заряда вдоль контура к этому заряду.

Основная и дополнительная литература по теме урока:

Обязательная литература:

1. Г.Я. Мякишев., Б.Б.Буховцев., Н.Н.Сотский. Физика.10 класс. Учебник для общеобразовательных организаций М.: Просвещение, 2017. – С. 343 – 347.

Рымкевич А.П. Сборник задач по физике. 10-11 класс. — М.: Дрофа,2009.- 68 – 74.

Дополнительная литература.

http://kvant.mccme.ru/1972/10/zakon_dzhoulya-lenca.htm

Основное содержание урока

При упорядоченном движении заряженных частиц в проводнике электрическое поле совершает работу, равную произведению заряда, прошедшего через проводник, и напряжения.

Сила тока равна отношению заряда прошедшего через проводник ко времени прохождения

Выразим заряд из формулы силы тока

через силу тока и время:

после подстановки в формулу (1) получим

Работа тока на участке цепи равна произведению силы тока, напряжения и времени, в течение которого шёл ток.

Из закона Ома для участка цепи выразим напряжение через силу тока и напряжение

и подставив в формулу работы получим:

При последовательном соединении проводников для определения работы тока удобнее пользоваться этой формулой, так как сила тока одинакова во всех проводниках.

При параллельном соединении проводников формулой:

так как напряжение на всех проводниках одинаково.

Работа тока показывает, сколько электроэнергии превратилось в другие виды энергии за конкретный период времени. Для электроэнергии справедлив закон сохранения энергии.

Мощность определяется по формуле:

Мощность тока равна отношению работы тока ко времени прохождения тока.

Так же формулу для мощности можно переписать в нескольких эквивалентных формах:

Если на участке цепи не совершается механическая работа и ток не производит химических действий, то происходит только нагревание проводника.

Электрическое поле действует с силой на свободные электроны, которые начинают упорядоченно двигаться, одновременно участвуя в хаотическом движении, ускоряясь в промежутках между столкновениями с ионами кристаллической решетки. Во время этих столкновений расходуется кинетическая энергия заряженных частиц. Именно эта энергия и становится теплом. Последующие столкновения электронов с другими ионами увеличивают амплитуду их колебаний и соответственно температуру всего проводника.

В неподвижных металлических проводниках вся работа тока идет на увеличение их внутренней энергии:

Количество теплоты, выделяемое проводником, по которому течет ток, равно работе тока.

Количество теплоты, выделяемое проводником с током, равно произведению квадрата силы тока, сопротивления проводника и времени прохождения тока по проводнику:

При последовательном соединении большее количество теплоты выделяется в проводнике с большим сопротивлением, а при параллельном соединении – с меньшим.

Измерения, приводящие к закону Джоуля-Ленца, можно выполнить, поместив в калориметр с водой проводник с известным сопротивлением и пропуская через него ток определенной силы в течение известного времени. Количество выделяющейся при этом теплоты определяют, составив уравнение теплового баланса.

Если соединить проводником два металлических шарика, несущих заряды противоположных знаков, под влиянием электрического поля этих зарядов в проводнике возникает кратковременный электрический ток. Заряды быстро нейтрализуют друг друга, и электрическое поле исчезнет.

Чтобы ток был постоянным, надо поддерживать постоянное напряжение между шариками. Для этого необходимо устройство, которое перемещало бы заряды от одного шарика к другому в направлении, противоположном направлению сил, действующих на эти заряды со стороны электрического поля шариков. В таком устройстве на заряды, должны действовать силы неэлектростатического происхождения. Одно лишь электрическое поле заряженных частиц не способно поддерживать постоянный ток в цепи.

Любые силы, действующие на электрически заряженные частицы, за исключением сил электростатического происхождения (то есть кулоновских), называют сторонними силами. Необходимости сторонних сил для поддержания постоянного тока в цепи объясняет закон сохранения энергии.

Электростатическое поле потенциально. Работа этого поля при перемещении в нем заряженных частиц вдоль замкнутой электрической цепи равна нулю. Прохождение же тока по проводникам сопровождается выделением энергии — проводник нагревается. Следовательно, в цепи должен быть какой-то источник энергии, поставляющий ее в цепь. Работа этих сил вдоль замкнутого контура отлична от нуля. Внутри источника тока заряды движутся под действием сторонних сил против кулоновских сил (электроны от положительно заряженного электрода к отрицательному), а во внешней цепи их приводит в движение электрическое поле.

Действие сторонних сил характеризуется важной физической величиной, называемой электродвижущей силой (сокращенно ЭДС).

Электродвижущая сила источника тока равна отношению работы сторонних сил при перемещении заряда по замкнутому контуру к величине этого заряда:

Электродвижущую силу выражают в вольтах.

Разбор тренировочных заданий

1. Электрочайник со спиралью нагревательного элемента сопротивлением 30 Ом включен в сеть напряжением 220 В. Какое количество теплоты выделится в нагревательном элемента за 5 мин?

1) 7260000 Дж;

2) 2200 Дж;

3) 484000 Дж.

Дано:

R=30Ом

U=220B

t=5мин=300с

Найти Q-?

Решение.2}Rdt\).

Выражение (5) и представляет собой закон Джоуля – Ленца, экспериментально установленный независимо друг от друга Дж. Джоулем и Э.X. Ленцем.

МОЩНОСТЬ ПОСТОЯННОГО ТОКА – это отношение работы тока за время \(t\) к этому интервалу времени.

\(P=\frac At = \frac{U\cdot q}t = \frac {U\cdot I \cdot t}t=UI\)

В системе СИ: \([P] = B \cdot A =\) Вт.

Репетитор-онлайн — подготовка к ЦТ

Пример 17. Электроплитка состоит из двух спиралей, соединенных параллельно и имеющих мощности 80 и 40 Вт. Известно, что плитка нагревает воду до кипения за 30 мин в том случае, когда спирали работают вместе. Во сколько раз увеличится время нагревания воды до кипения, если первая спираль перегорит через 15 мин после включения?

Решение. Пусть на нагревание воды до кипения требуется теплота Q. Указанное количество теплоты по условию задачи вода получает двумя путями:

  • в первом случае от двух одновременно работающих спиралей:

Q = Q 1 + Q 2,

где Q 1 — количество теплоты, выделяемое первой спиралью, Q 1 = = P 1t 0; P 1 — мощность первой спирали; t 0 — время работы плитки, t 0 = 30 мин; Q 2 — количество теплоты, выделяемое второй спиралью, Q 2 = P 2t 0; P 2 — мощность второй спирали;

  • во втором случае от двух спиралей, работающих разное время:

Q=Q′1+Q′2,

где Q′1 — количество теплоты, выделяемое первой спиралью, Q′1=P1t1; t 1 — время работы первой спирали, t 1 = 15 мин; Q′2 — количество теплоты, выделяемое второй спиралью, Q′2=P2t2; t 2 — время работы второй спирали.

Равенства, записанные в явном виде, образуют систему уравнений:

Q=P1t0+P2t0,Q=P1t1+P2t2.}

Величина t 2 представляет собой время работы плитки во втором случае.

Для того чтобы найти указанную величину, приравняем правые части уравнений системы:

P1t0+P2t0=P1t1+P2t2.

Отсюда следует:

t2=(P1+P2)t0−P1t1P2=(P1P2+1)t0−P1P2t1.

Искомое отношение определяется формулой

t2t0=(P1P2+1)−P1P2t1t0.

Вычислим:

t2t0=(8040+1)−80⋅1540⋅30=2,0.

Время нагревания воды до кипения увеличится в 2,0 раза.

Закон Джоуля-Ленца — формула, применение на практике, вывод

Этот урок посвящён изучению теплового действия электрического тока. Мы проведём ряд опытов, демонстрирующих зависимость количества теплоты от силы тока и сопротивления, а также  рассмотрим закон Джоуля – Ленца

Блок: 1/8 | Кол-во символов: 209
Источник: https://interneturok.ru/lesson/physics/8-klass/belektricheskie-yavleniyab/nagrevanie-provodnikov-elektricheskim-tokom-zakon-dzhoulya-lentsa-grebenyuk-yu-v?konspekt=

Введение

Мы уже знаем, что при прохождении тока через электрическую лампочку её спираль нагревается и излучает видимый свет. Таким образом, мы наблюдаем тепловое действие электрического тока. Благодаря этому действию, нагреваются, например, утюг или чайник. Но при работе вентилятора или пылесоса практически не наблюдается тепловое действие, также в нормальном состоянии слабо греются провода. На этом уроке, тема которого: «Нагревание проводников электрическим током. Закон Джоуля – Ленца», мы определим, от чего зависит тепловое действие электрического тока.

Блок: 2/8 | Кол-во символов: 562
Источник: https://interneturok.ru/lesson/physics/8-klass/belektricheskie-yavleniyab/nagrevanie-provodnikov-elektricheskim-tokom-zakon-dzhoulya-lentsa-grebenyuk-yu-v?konspekt=

Определения

В словесной формулировке звучит следующим образом:

Мощность тепла, выделяемого в единице объёма среды при протекании постоянного электрического тока, равна произведению плотности электрического тока на величину напряженности электрического поля.

Математически может быть выражен в следующей форме:

где  — мощность выделения тепла в единице объёма,  — плотность электрического тока,  — напряжённость электрического поля, σ — проводимость среды, а точкой обозначено скалярное произведение.

Закон также может быть сформулирован в интегральной форме для случая протекания токов в тонких проводах:

Количество теплоты, выделяемое в единицу времени в рассматриваемом участке цепи, пропорционально произведению квадрата силы тока на этом участке и сопротивления участка.

В интегральной форме этот закон имеет вид

где  — количество теплоты, выделяемое за промежуток времени ,  — сила тока,  — сопротивление,  — полное количество теплоты, выделенное за промежуток времени от до . В случае постоянных силы тока и сопротивления:

Применяя закон Ома, можно получить следующие эквивалентные формулы:

Блок: 2/5 | Кол-во символов: 1115
Источник: https://ru.wikipedia.org/wiki/%D0%97%D0%B0%D0%BA%D0%BE%D0%BD_%D0%94%D0%B6%D0%BE%D1%83%D0%BB%D1%8F_%E2%80%94_%D0%9B%D0%B5%D0%BD%D1%86%D0%B0

Физический смысл закона Джоуля – Ленца

Закон можно объяснить следующим образом: ток, протекая по проводнику, представляет собой перемещение электрического заряда под воздействием электрического поля. Таким образом, электрическое поле совершает некоторую работу. Эта работа расходуется на нагрев проводника.

Другими словами, энергия переходит в другое свое качество – тепло.

Но чрезмерный нагрев проводников с током и электрооборудования допускать нельзя, поскольку это может привести к их повреждению. Опасен сильный перегрев при коротких замыканиях проводов, когда по проводниках могут протекать достаточно большие токи.

В интегральной форме для тонких проводников закон Джоуля – Ленца звучит следующим образом: количество теплоты, которое выделяется в единицу времени в рассматриваемом участке цепи, определяется как произведение квадрата силы тока на сопротивление участка.

Математически эта формулировка выражается следующим образом:

Q = ∫ k • I² • R • t,

при этом Q – количество выделившейся теплоты;

I – величина тока;

R — активное сопротивление проводников;

t – время воздействия.

Значение параметра k принято называть тепловым эквивалентом работы. Величина этого параметра определяется в зависимости от разрядности единиц, в которых выполняются измерения значений, используемых в формуле.

Закон Джоуля-Ленца имеет достаточно общий характер, поскольку не имеет зависимости от природы сил, генерирующих ток.

Из практики можно утверждать, что он справедлив, как для электролитов, так проводников и полупроводников.

Блок: 3/4 | Кол-во символов: 1521
Источник: https://pue8.ru/elektrotekhnik/823-zakon-dzhoulya-lentsa-opredelenie-formula-fizicheskij-smysl.html

Формула Джоуля-Ленца

В итоге, спустя десятилетие, в 1843 году Эмилий Ленц выставил на  всеобщее обозрение научного сообщества результат своих опытов в виде закона. Однако, оказалось, что его опередили! Пару лет назад английский физик Джеймс Прескотт Джоуль уже проводил аналогичные опыты и также представил общественности свои результаты. Но, тщательно проверив все работы Джеймса Джоуля, русский учёный выяснил что собственные опыты гораздо точнее, наработан больший объём исследований, потому, русской науке есть чем дополнить английское открытие.

Научное сообщество рассмотрело оба результата исследований и объединила их в одно, тем самым закон Джоуля переименовали в закон Джоуля-Ленца. Закон утверждает, что количество теплоты, выделяемое проводником при протекании по нему электрического тока , равно произведению силы этого тока в квадрате, сопротивлению проводника и времени, за которое по проводнику течёт ток. Или формулой:

Q=I2Rt

где

Q — количество выделяемого тепла (Джоули)

I — сила тока, протекающего через проводник (Амперы)

R — сопротивление проводника (Омы)

t — время прохождения тока через проводник (Секунды)

Блок: 3/6 | Кол-во символов: 1135
Источник: https://www.RusElectronic.com/zakon-dzhoulja-ljentsa/

Применение и практический смысл

Непосредственное превращение электричества в тепловую энергию нельзя назвать экономически выгодным. Однако, с точки зрения удобства и доступности современного человечества к источникам электроэнергии различные нагревательные приборы продолжают массово применяться как в быту, так и на производстве.

Перечислим некоторые из них:

  • электрочайники;
  • утюги;
  • фены;
  • варочные плиты;
  • паяльники;
  • сварочные аппараты и многое другое.

На рисунке 3 изображены бытовые нагревательные приборы, которыми мы часто пользуемся.

Рис. 3. Бытовые нагревательные приборы

Использование тепловых мощностей в химической, металлургической и в других промышленных отраслях тесно связно с использованием электрической энергии.

Без знания физического закона Джоуля-Ленца было бы невозможно сконструировать безопасный нагревательный прибор. Для этого нужны расчёты, которые невозможно сделать без применения рассмотренных нами формул. На основе расчётов происходит выбор материалов с нужным удельным сопротивлением, влияющим на нагревательную способность устройств.

Закон Джоуля-Ленца без преувеличения можно назвать гениальным. Это один из тех законов, которые повлияли на развитие электротехники.

Блок: 4/4 | Кол-во символов: 1209
Источник: https://www.asutpp.ru/zakon-dzhoulya-lentsa.html

Почему греется проводник

Как же объясняется нагрев проводника? Почему он именно греется, а не остаётся нейтральным или охлаждается? Нагрев происходит из-за того, что свободные электроны, перемещающиеся в проводнике под действием электрического поля, бомбардируют атомы молекул металла, тем самым передавая им собственную энергию, которая переходит в тепловую. Если изъясняться совсем просто: преодолевая материал проводника, электрический ток как бы “трётся”, соударяется электронами о молекулы проводника. Ну а , как известно, любое трение сопровождается нагревом. Следовательно, проводник будет нагреваться пока по нему бежит электрический ток.

Из формулы также следует –  чем выше удельное сопротивление проводника и чем выше сила тока протекающего по нему, тем выше будет нагрев . Например, если последовательно соединить проводник-медь (удельное сопротивление  0,018 Ом·мм²/м) и проводник-алюминий (0,027 Ом·мм²/м), то при протекании через цепь электрического тока алюминий будет нагреваться сильнее чем медь из-за более высокого сопротивления. Поэтому, кстати, не рекомендуется в быту делать скрутки медных и алюминиевых проводов друг с другом – будет неравномерный нагрев в месте скрутки. В итоге –  подгорание с последующим пропаданием контакта.

Блок: 4/6 | Кол-во символов: 1255
Источник: https://www.RusElectronic.com/zakon-dzhoulja-ljentsa/

Опыты Ленца

Перенесемся в 19 век-эпоху накопления знаний и подготовки к технологическому прыжку 20 века. Эпоха, когда по всему миру различные учёные и просто изобретатели-самоучки чуть ли не ежедневно открывают что-то новое, зачастую тратя огромное количество времени на исследования и, при этом, не представляя конечный результат.

Один из таких людей, русский учёный Эмилий Христианович Ленц, увлекался электричеством, на тогдашнем примитивном уровне, пытаясь рассчитывать  электрические цепи. В 1832 году  Эмилий Ленц “застрял” с расчётами, так как параметры его смоделированной цепи “источник энергии – проводник – потребитель энергии” сильно разнились от опыта к опыту. Зимой 1832-1833 года учёный обнаружил, что причиной нестабильности является кусочек платиновой проволоки, принесённый им с холода. Отогревая или охлаждая проводник, Ленц также заметил что  существует некая  зависимость между силой тока, электрическим сопротивлением  и температурой проводника.

При определённых параметрах электрической цепи проводник быстро оттаивал и даже слегка нагревался. Измерительных приборов в те времена практически никаких не существовало – невозможно было точно измерить ни силу тока, ни сопротивление. Но это был русский физик, и он проявил смекалку. Если это зависимость, то почему бы ей не быть обратимой?

Для того чтобы измерить количество тепла, выделяемого проводником, учёный сконструировал простейший “нагреватель” – стеклянная ёмкость, в которой находился  спиртосодержащий раствор и погружённый в него платиновый проводник-спираль. Подавая различные величины электрического тока на проволоку, Ленц замерял время, за которое раствор нагревался до определённой температуры. Источники электрического тока в те времена  были слишком слабы, чтобы разогреть раствор до серьёзной температуры, потому визуально определить количество испарившегося  раствора не представлялось возможным. Из-за этого процесс исследования очень затянулся – тысячи вариантов подбора параметров источника питания, проводника, долгие замеры и последующий анализ.

Блок: 2/6 | Кол-во символов: 2044
Источник: https://www.RusElectronic.com/zakon-dzhoulja-ljentsa/

Итоги урока

На этом уроке мы узнали о том, что прохождение тока в проводнике сопровождается выделением тепла, при этом количество теплоты, выделяющееся при прохождении тока в проводнике, прямо пропорционально квадрату силы тока, сопротивлению проводника и времени прохождения тока. Таким образом, мы сформулировали закон Джоуля – Ленца.

Блок: 5/8 | Кол-во символов: 337
Источник: https://interneturok.ru/lesson/physics/8-klass/belektricheskie-yavleniyab/nagrevanie-provodnikov-elektricheskim-tokom-zakon-dzhoulya-lentsa-grebenyuk-yu-v?konspekt=

Применение закона Джоуля-Ленца в жизни

Открытие закона Джоуля-Ленца имело огромные последствия для практического применения электрического тока. Уже в 19 веке стало возможным создать более точные измерительные приборы, основанные на сокращении проволочной спирали при её нагреве протекающим током определённой величины – первые стрелочные вольтметры и амперметры. Появились первые прототипы электрических обогревателей, тостеров, плавильных печей – использовался проводник с высоким удельным сопротивлением, что позволяло получить довольно высокую температуру.

Были изобретены плавкие предохранители, биметаллические прерыватели цепи (аналоги современных тепловых реле защиты), основанные на разнице нагрева проводников с разным удельным сопротивлением. Ну и, конечно же, обнаружив что при определённой силе тока проводник с высоким удельным сопротивлением способен нагреться докрасна , данный эффект использовали в качестве источника света. Появились первые лампочки.

Проводник (угольная палочка, бамбуковая нить, платиновая проволока и т.д.) помещали в стеклянную  колбу, откачивали воздух для замедления процесса окисления и получали  незатухаемый, чистый и стабильный источник света – электрическую лампочку

Блок: 5/6 | Кол-во символов: 1219
Источник: https://www.RusElectronic.com/zakon-dzhoulja-ljentsa/

Задача из ЕГЭ

По проводнику сопротивлением R течёт ток I. Как изменится количество теплоты, выделяющееся в проводнике в единицу времени, если его сопротивление увеличить в два раза, а силу тока уменьшить в два раза? Варианты ответа: а) увеличится в два раза; б) уменьшится в два раза; в) не изменится; г) уменьшится в восемь раз.

Решение

Воспользуемся законом Джоуля – Ленца:

 

Количество теплоты, выделяющееся в проводнике в единицу времени, равно:

 

Так как сопротивление увеличивается в два раза, а сила тока уменьшается в два раза:

 

 

Следовательно, новое значение количества теплоты будет равно:

 

  

Ответ: б) уменьшится в два раза

Блок: 6/8 | Кол-во символов: 651
Источник: https://interneturok.ru/lesson/physics/8-klass/belektricheskie-yavleniyab/nagrevanie-provodnikov-elektricheskim-tokom-zakon-dzhoulya-lentsa-grebenyuk-yu-v?konspekt=

Заключение

Таки образом, можно сказать что на законе Джоуля-Ленца держится чуть ли не вся электрика и электротехника. Открыв этот закон, появилась возможность уже заранее предсказать  некоторые будущие проблемы в освоении электричества. Например, из-за нагрева проводника передача электрического тока на большое расстояние сопровождается потерями этого тока на тепло. Соответственно, чтобы компенсировать эти потери  нужно занизить передаваемый ток, компенсируя это высоким напряжением. А уже на оконечном потребителе, понижать напряжение и получать более высокий ток.

Закон Джоуля-Ленца неотступно следует из одной эпохи технологического развития  в другую. Даже сегодня мы постоянно наблюдаем его в быту – закон проявляется всюду, и не всегда люди ему рады. Сильно греющийся процессор персонального компьютера, пропадание света из-за обгоревшей скрутки  «медь-алюминий»,выбитая вставка-предохранитель, выгоревшая из-за высокой нагрузки электропроводка – всё это тот самый закон Джоуля-Ленца.

Блок: 6/6 | Кол-во символов: 993
Источник: https://www.RusElectronic.com/zakon-dzhoulja-ljentsa/

Решение задач

Задача 1

Определите длину нихромового провода, с площадью сечения 0,25 , из которого изготовлен нагреватель электрического чайника. Чайник питается от сети напряжением 220 В и нагревает 1,5 литра воды от  до  за 10 минут. КПД чайника составляет .

Дано: ; ; ; ; ; ;  – теплоёмкость воды;  – плотность воды;  – удельное сопротивление нихрома;  

Найти:l

Решение

Так как вся электрическая энергия идёт на нагревание воды, то воспользуемся законом Джоуля – Ленца:

 

Отсюда сопротивление проводника (нихромового провода) Rравно:

 

Также сопротивление проводника можно вычислить по формуле:

 

Приравняем сопротивление в обеих формулах и выразим длину проводника (l):

  

  

В этой формуле неизвестно количество теплоты, то есть мощность чайника. Найдём её, зная, что чайник нагревает 1,5 л воды от от  до  за 10 минут.

 

 

Так как не вся теплота идёт на нагревание, то необходимо учитывать КПД чайника, равный:

 

Отсюда общее количество теплоты () будет равно:

 

Подставим значение   в формулу для длины проводника:

 

Проверив единицы измерения, подставляем известные значения:

 

Ответ:  

Задача 2

С какой целью провода в местах соединения не просто скручивают, но ещё и спаивают? Ответ обоснуйте.

Решение

Рис. 4. Иллюстрация к задаче

Сила тока в обоих проводах одинакова, так как проводники соединены последовательно (см. Рис. 4):

 

 

Если место контакта двух проводников не будет спаяно, то его сопротивление будет достаточно большое, по сравнению с сопротивлением самих проводников. Следовательно, в месте контакта будет выделяться наибольшее количество теплоты, что приведёт к расплавлению места контакта и размыканию электрической цепи. Поэтому провода в местах соединения не просто скручивают, но ещё и спаивают с целью уменьшения сопротивления.

Задача 3

Какой длины нихромовый провод нужно взять, чтобы изготовить электрический камин, работающий при напряжении 120 В и выделяющий 1 МДж теплоты в час? Диаметр провода 0,5 мм.

Дано: ; ; ;  ;

Найти:l

Решение

Так как вся электрическая энергия расходуется на нагревание, то согласно закону Джоуля-Ленца:

 

Отсюда сопротивление провода равно:

 

Также сопротивление проводника можно вычислить по формуле:

 

Приравняем сопротивление в обеих формулах и выразим длину проводника (l):

 

 

В этой формуле неизвестна площадь сечения проволоки. Зная диаметр проволоки, вычислим площадь сечения проволоки по формуле площади круга:

  

Подставим значение в формулу для длины проводника:

 

Проверив единицы измерения, подставляем известные значения:

 

Ответ:

Список литературы

  1. Генденштейн Л.Э, Кайдалов А.Б., Кожевников В.Б. / Под ред. Орлова В.А., Ройзена И.И. Физика 8. – М.: Мнемозина.
  2. Перышкин А.В. Физика 8. – М.: Дрофа, 2010.
  3. Фадеева А.А., Засов А.В., Киселев Д.Ф. Физика 8. – М.: Просвещение.

Домашнее задание

  1. В чем проявляется тепловое действие тока?
  2. Как можно объяснить нагревание проводника с током?
  3. Известно, что безопасным для человека является постоянный ток 100 мкА. Какое количество теплоты выделится за 1 мин в теле человека при прохождении тока от конца одной руки до конца другой руки (при сухой коже), если сопротивление этого участка равно 15000 Ом?
  4. Участок цепи состоит из двух резисторов сопротивлением 8 Ом каждый, соединенных параллельно. Сила тока в цепи – 0,3 А. Какое количество теплоты выделится в участке за 1 мин?
  5. Сколько электроэнергии потребляет электрический утюг за 4 ч работы, если он включен в сеть напряжением 220 В при силе тока 4,55 А?

Дополнительные рекомендованные ссылки на ресурсы сети Интернет

Блок: 8/8 | Кол-во символов: 3560
Источник: https://interneturok.ru/lesson/physics/8-klass/belektricheskie-yavleniyab/nagrevanie-provodnikov-elektricheskim-tokom-zakon-dzhoulya-lentsa-grebenyuk-yu-v?konspekt=

Кол-во блоков: 20 | Общее кол-во символов: 20988
Количество использованных доноров: 5
Информация по каждому донору:
  1. https://ru.wikipedia.org/wiki/%D0%97%D0%B0%D0%BA%D0%BE%D0%BD_%D0%94%D0%B6%D0%BE%D1%83%D0%BB%D1%8F_%E2%80%94_%D0%9B%D0%B5%D0%BD%D1%86%D0%B0: использовано 2 блоков из 5, кол-во символов 3640 (17%)
  2. https://pue8.ru/elektrotekhnik/823-zakon-dzhoulya-lentsa-opredelenie-formula-fizicheskij-smysl.html: использовано 2 блоков из 4, кол-во символов 1809 (9%)
  3. https://www.asutpp.ru/zakon-dzhoulya-lentsa.html: использовано 1 блоков из 4, кол-во символов 1209 (6%)
  4. https://interneturok.ru/lesson/physics/8-klass/belektricheskie-yavleniyab/nagrevanie-provodnikov-elektricheskim-tokom-zakon-dzhoulya-lentsa-grebenyuk-yu-v?konspekt=: использовано 6 блоков из 8, кол-во символов 7684 (37%)
  5. https://www.RusElectronic.com/zakon-dzhoulja-ljentsa/: использовано 5 блоков из 6, кол-во символов 6646 (32%)

Закон джоуля ленца. Закон джоуля-ленца Количество теплоты через сопротивление и напряжение

Содержание:

Знаменитый русский физик Ленц и английский физик Джоуль, проводя опыты по изучению тепловых действий электрического тока, независимо друг от друга вывели закон Джоуля-Ленца. Данный закон отражает взаимосвязь количества теплоты, выделяемого в проводнике, и электрического тока, проходящего по этому проводнику в течение определенного периода времени.

Свойства электрического тока

Когда электрический ток проходит через металлический проводник, его электроны постоянно сталкиваются с различными посторонними частицами. Это могут быть обычные нейтральные молекулы или молекулы, потерявшие электроны. Электрон в процессе движения может отщепить от нейтральной молекулы еще один электрон. В результате, его кинетическая энергия теряется, а вместо молекулы происходит образование положительного иона. В других случаях электрон, наоборот, соединиться с положительным ионом и образовать нейтральную молекулу.

В процессе столкновений электронов и молекул происходит расход энергии, в дальнейшем превращающейся в тепло. Затраты определенного количества энергии связаны со всеми движениями, во время которых приходится преодолевать сопротивление. В это время происходит превращение работы, затраченной на преодоление сопротивления трения, в тепловую энергию.

Закон джоуля Ленца формула и определение

Согласно закону джоуля Ленца, электрический ток, проходящий по проводнику, сопровождается количеством теплоты, прямо пропорциональным квадрату тока и сопротивлению, а также времени течения этого тока по проводнику.

В виде формулы закон Джоуля-Ленца выражается следующим образом: Q = I 2 Rt, в которой Q отображает количество выделенной теплоты, I — , R — сопротивление проводника, t — период времени. Величина «к» представляет собой тепловой эквивалент работы и применяется в тех случаях, когда количество теплоты измеряется в калориях, сила тока — , сопротивление — в Омах, а время — в секундах. Численное значение величины к составляет 0,24, что соответствует току в 1 ампер, который при сопротивлении проводника в 1 Ом, выделяет в течение 1 секунды количество теплоты, равное 0,24 ккал. Поэтому для расчетов количества выделенной теплоты в калориях применяется формула Q = 0,24I 2 Rt.

При использовании системы единиц СИ измерение количества теплоты производится в джоулях, поэтому величина «к», применительно к закону Джоуля-Ленца, будет равна 1, а формула будет выглядеть: Q = I 2 Rt. В соответствии с I = U/R. Если это значение силы тока подставить в основную формулу, она приобретет следующий вид: Q = (U 2 /R)t.

Основная формула Q = I 2 Rt очень удобна для использования при расчетах количества теплоты, которое выделяется в случае последовательного соединения. Сила тока во всех проводниках будет одинаковая. При последовательном соединении сразу нескольких проводников, каждый из них выделит столько теплоты, которое будет пропорционально сопротивлению проводника. Если последовательно соединить три одинаковые проволочки из меди, железа и никелина, то максимальное количество теплоты будет выделено последней. Это связано с наибольшим удельным сопротивлением никелина и более сильным нагревом этой проволочки.

При параллельном соединении этих же проводников, значение электрического тока в каждом из них будет различным, а напряжение на концах — одинаковым. В этом случае для расчетов больше подойдет формула Q = (U 2 /R)t. Количество теплоты, выделяемое проводником, будет обратно пропорционально его проводимости. Таким образом, закон Джоуля — Ленца широко используется для расчетов установок электрического освещения, различных отопительных и нагревательных приборов, а также других устройств, связанных с преобразованием электрической энергии в тепловую.

Закон Джоуля-Ленца. Работа и мощность электрического тока

Задача по теме «Законы постоянного тока». Задача может быть интересна учащимся 10-х классов и выпускникам для подготовки к ЕГЭ. Кстати, подобного рода задача была на ЕГЭ в части 1 с несколько иным вопросом (необходимо было найти отношение количеств теплоты, выделяющихся на резисторах).

На каком из резисторов выделится наибольшее (наименьшее) количество теплоты? R1 = R4 = 4 Ом, R2 = 3 Ом, R3 = 2Ом. Дать решение. Чтобы ответить на вопрос задачи, необходимо сравнить количество теплоты, выделяющееся на каждом их резисторов. Для этого воспользуемся формулой закона Джоуля — Ленца. То есть основной задачей будет являться определение силы тока (или сравнение), протекающей через каждый резистор.

Согласно законам последовательного соединения, сила тока, протекающая через резисторы R1 и R2, и R3 и R4, одинаковая.Чтобы определить силу тока в верхней и в нижней ветвях, воспользуемся законом параллельного соединения, согласно которому, напряжение на этих ветвях одинаковое.Расписывая напряжение на нижней и верхней ветвях по закону Ома для участка цепи, имеем: Подставляя численные значения сопротивлений резисторов, получаем:То есть получаем соотношение между токами, протекающими в верхней и в нижней ветви:Определив силу тока через каждый из этих резисторов, определяем количество теплоты, выделяющееся на каждом из резисторов.Сравнивая числовые коэффициенты, приходим к выводу, что максимальное количество теплоты выделится на четвёртом резисторе, а минимальное количество теплоты — на втором.

Вы можете оставить комментарий, или поставить трэкбек со своего сайта.

Написать комментарий

fizika-doma.ru

Тепловая мощность — формула расчета

С теплотехническими расчётами приходится сталкиваться владельцам частных домов, квартир или любых других объектов. Это основа основ проектирования зданий.

Понять суть этих расчётов в официальных бумагах, не так сложно, как кажется.

Для себя также можно научиться выполнять вычисления, чтобы решить, какой утеплитель применять, какой толщины он должен быть, какой мощности приобретать котёл и достаточно ли имеющихся радиаторов на данную площадь.

Ответы на эти и многие другие вопросы можно найти, если понять, что такое тепловая мощность. Формула, определение и сферы применения – читайте в статье.

Что такое тепловой расчет?

Если говорить просто, тепловой расчёт помогает точно узнать, сколько тепла хранит и теряет здание, и сколько энергии должно вырабатывать отопление, чтобы поддерживать в жилье комфортные условия.

Оценивая теплопотери и степень теплоснабжения, учитываются следующие факторы:

  1. Какой это объект: сколько в нём этажей, наличие угловых комнат, жилой он или производственный и т. д.
  2. Сколько человек будет «обитать» в здании.
  3. Важная деталь — это площадь остекления. И размеры кровли, стен, пола, дверей, высота потолков и т. д.
  4. Какова продолжительность отопительного сезона, климатические характеристики региона.
  5. По СНиПам определяют нормы температур, которые должны быть в помещениях.
  6. Толщина стен, перекрытий, выбранные теплоизоляторы и их свойства.

Могут учитываться и другие условия и особенности, например, для производственных объектов считаются рабочие и выходные дни, мощность и тип вентиляции, ориентация жилья по сторонам света и др.

Для чего нужен тепловой расчет?

Как умудрялись обходиться без тепловых расчётов строители прошлого?

Сохранившиеся купеческие дома показывают, что всё делалось просто с запасом: окна поменьше, стены — потолще. Получалось тепло, но экономически не выгодно.

Теплотехнический расчёт позволяет строить наиболее оптимально. Материалов берётся ни больше — ни меньше, а ровно столько, сколько нужно. Сокращаются габариты строения и расходы на его возведение.

Вычисление точки росы позволяет строить так, чтобы материалы не портились как можно дольше.

Для определения необходимой мощности котла также не обойтись без расчётов. Суммарная мощность его складывается из затрат энергии на обогрев комнат, нагрев горячей воды для хозяйственных нужд, и способности перекрывать теплопотери от вентиляции и кондиционирования. Прибавляется запас мощности, на время пиковых холодов.

При газификации объекта требуется согласование со службами. Рассчитывается годовой расход газа на отопление и общая мощность тепловых источников в гигакалориях.

Нужны расчёты при подборе элементов отопительной системы. Обсчитывается система труб и радиаторов – можно узнать, какова должна быть их протяжённость, площадь поверхности. Учитывается потеря мощности при поворотах трубопровода, на стыках и прохождении арматуры.

При расчетах затрат тепловой энергии могут пригодиться знания, как перевести Гкал в Квт и обратно. В следующей статье подробно рассмотрена эта тема с примерами расчета.

Полный расчет теплого водяного пола приведен в этом примере.

Знаете ли вы, что количество секций радиаторов отопления не берется «с потолка»? Слишком малое их количество приведет к тому, что в доме будет холодно, а чрезмерно больше создаст жару и приведет к чрезмерной сухости воздуха. По ссылке http://microklimat.pro/sistemy-otopleniya/raschet-sistem-otopleniya/kolichestva-sekcij-radiatorov.html приведены примеры правильного расчета радиаторов.

Расчет тепловой мощности: формула

Рассмотрим формулу и приведем примеры, как произвести расчет для зданий с разным коэффициентом рассеивания.

Vx(дельта)TxK= ккал/ч (тепловая мощность), где:

  • Первый показатель «V» – объем рассчитываемого помещения;
  • Дельта «Т» — разница температур – это та величина, которая показывает насколько градусов внутри помещения теплее, чем снаружи;
  • «К» — коэффициент рассеивания (его еще называют «коэффициент пропускания тепла»). Величина берется из таблицы. Обычно цифра колеблется от 4 до 0,6.
Примерные величины коэффициента рассеивания для упрощенного расчёта
  • Если это неутепленный металлопрофиль или доска то «К» будет = 3 – 4 единицы.
  • Одинарная кирпичная кладка и минимальное утепление – «К» = от 2 до 3-ёх.
  • Стена в два кирпича, стандартное перекрытие, окна и
  • двери – «К» = от 1 до 2.
  • Самый теплый вариант. Стеклопакеты, кирпичные стены с двойным утеплителем и т. п. – «К» = 0,6 – 0,9.

Более точный расчет можно произвести, высчитывая точные размеры отличающихся по свойствам поверхностей дома в м2 (окна, двери и т. д.), производя расчёт для них отдельно и складывая получившиеся показатели.

Пример расчета тепловой мощности

Возьмем некое помещение 80 м2 с высотой потолков 2,5 м и посчитаем, какой мощности котел нам потребуется для его отопления.

Вначале высчитываем кубатуру: 80 х 2,5 = 200 м3. Дом у нас утеплен, но недостаточно – коэффициент рассеивания 1,2.

Морозы бывают до -40 °C, а в помещении хочется иметь комфортные +22 градуса, разница температур (дельта «Т») получается 62 °C.

Подставляем в формулу мощности тепловых потерь цифры и перемножаем:

200 х 62 х 1,2 = 14880 ккал/ч.

Полученные килокалории переводим в киловатты, пользуясь конвертером:

  • 1 кВт = 860 ккал;
  • 14880 ккал = 17302,3 Вт.

Округляем в большую сторону с запасом, и понимаем, что в самый сильный мороз -40 градусов нам потребуется 18 кВт энергии в час.

Умножаем периметр дома на высоту стен:

(8 + 10) х 2 х 2,5 = 90 м2 поверхности стены + 80 м2 потолок = 170 м2 поверхности, контактирующей с холодом. Теплопотери, высчитанные нами выше, составили 18 кВт/ч, делим поверхность дома на расчетную израсходованную энергию получаем, что 1 м2 теряет примерно 0,1 кВт или 100 Вт ежечасно при температуре на улице -40 °C, а в помещении +22 °С.

Эти данные могут стать основой для расчёта требуемой толщины утеплителя на стены.

Приведем другой пример расчета, он в некоторых моментах сложнее, но более точный.

Формула:

Q = S x (дельта)T / R:

  • Q– искомая величина теплопотерь дома в Вт;
  • S– площадь охлаждающих поверхностей в м2;
  • T– разница температур в градусах Цельсия;
  • R– тепловое сопротивление материала (м2 х К/Вт) (Метры квадратные умноженные на Кельвин и делёный на Ватт).

Итак, чтобы найти «Q» того же дома, что и в примере выше, подсчитаем площадь его поверхностей «S» (пол и окна считать не будем).

  • «S» в нашем случае = 170 м2, из них 80 м2 потолок и 90 м2 — стены;
  • T = 62 °С;
  • R– тепловое сопротивление.

Ищем «R» по таблице тепловых сопротивлений или по формуле. Формула для расчета по коэффициенту теплопроводности такая:

R= H/ К.Т. (Н – толщина материала в метрах, К.Т. – коэффициент теплопроводности).

В этом случае, дом у нас имеет стены в два кирпича обшитые пенопластом толщиной 10 см. Потолок засыпан опилками толщиной 30 см.

Отопительную систему частного дома нужно устраивать с учетом экономии средств на энергоносители. Расчет системы отопления частного дома, а также рекомендации по выбору котлов и радиаторов — читайте внимательно.

Чем и как утеплить деревянный дом изнутри, вы узнаете, прочитав эту информацию. Выбор утеплителя и технология утепления.

Из таблицы коэффициентов теплопроводности (измеряется Вт / (м2 х К) Ватт делёный на произведение метра квадратного на Кельвин). Находим значения для каждого материала, они будут:

  • кирпич — 0,67;
  • пенопласт – 0,037;
  • опилки – 0,065.
Подставляем данные в формулу (R= H/ К.Т.):
  • R (потолка 30 см толщиной) = 0,3 / 0,065 = 4,6 (м2 х К) / Вт;
  • R (кирпичной стены 50 см) = 0,5 / 0,67 = 0,7 (м2 х К) / Вт;
  • R (пенопласт 10 см) = 0,1 / 0,037 = 2,7 (м2 х К) / Вт;
  • R (стен) = R(кирпич) + R(пенопласт) = 0,7 + 2,7 = 3,4 (м2 х К) / Вт.

Теперь можем приступить к расчету теплопотерь «Q»:

  • Q для потолка = 80 х 62 / 4,6 = 1078,2 Вт.
  • Q стен = 90 х 62 / 3,4 = 1641,1 Вт.
  • Остается сложить 1078,2 + 1641,1 и перевести в кВт, получается (если сразу округлить) 2,7 кВт энергии за 1 час.
Можно обратить внимание, насколько большая разница получилась в первом и втором случае, хотя объём домов и температура за окном в первом и втором случае были совершенно одинаковыми.

Всё дело в степени утомлённости домов (хотя, конечно, данные могли быть и иными, если бы мы рассчитывали пол и окна).

Заключение

Приведённые формулы и примеры показываю, что при теплотехнических расчётах очень важно учитывать как можно больше факторов, влияющих на теплопотери. Сюда входит и вентиляция, и площадь окон, степень их утомлённости и т. д.

А подход, когда на 10 м2 дома берётся 1 кВт мощности котла – слишком приблизительный, чтобы всерьёз опираться на него.

Видео на тему

microklimat.pro

13 Тепловой расчет

10. Тепловой расчет.

Конструкция ИМС должна быть такой, чтобы теплота, выделяющаяся при ее функционировании, не приводила в наиболее неблагоприятных условиях эксплуатации к отказам элементов в результате перегрева. К основным тепловыделяющим элементам следует отнести, прежде всего, резисторы, активные элементы и компоненты. Мощности, рассеиваемые конденсаторами и индуктивностями, невелики. Пленочная коммутация ИМС, благодаря малому электрическому сопротивлению и высокой теплопроводности металлических пленок, способствует отводу теплоты от наиболее нагретых элементов и выравниванию температуры платы ГИС и кристалла полупроводниковой ИМС.

Рис. 10.1. Вариант крепления платы на корпус.

Тепловой расчёт резисторов.

Тепловое сопротивление резистора вычислим по формуле (10.1)

п = 0.03 [Вт/см °С] — коэффициент теплопроводности материала подложки;

δп = 0.06 см – толщина платы.

RT=0.06/0.03=2 см2∙°С/Вт

Рассчитаем температуру пленочных резисторов по формуле

PR – мощность, выделяемая на резисторе;

SR – площадь, занимаемая резистором на плате;

P0 – суммарная мощность, выделяемая всеми компонентами микросхемы;

Sп – площадь платы.

PR = 0.43 мВт – мощность выделяемая на резисторе;

SR = 0.426мм2 – площадь занимаемая резистором;

Sn = 80 мм2 – площадь платы;

RT = 2 см2∙°С/Вт – тепловое сопротивление резистора;

Токр.ср = 40С – максимальная температура окружающей среды;

T = 125С = максимально допустимая температура пленочных резисторов.

TR=(0.43∙10-3∙200)/0.426+(24.82∙10-3∙200)/80+40=40.26 С

Температура остальных резисторов рассчитывается аналогично с помощью программы MathCad. Результаты расчётов представлены в Таблице10.1

Таблица. 10.1

Из таблицы видно, что для всех пленочных резисторов заданный тепловой режим соблюдается.

Тепловой расчет для навесного элемента.

Тепловое сопротивление будет вычисляться по формуле:

k = 0.003 [Вт/см °С] — коэффициент теплопроводности клея;

δк1 = 0.01 см – толщина клея.

Rт=(0.06/0.03)+(0.01/0.003)=5.33 см2∙°С/Вт

Рассчитаем температуру навесного элемента по формуле:

Расчет транзистор КТ202А, VT14

Pнэ = 2,6 мВт – мощность выделяемая на транзисторе;

Sнэ = 0,49 мм2 – площадь занимаемая транзистором;

P0 = 24.82 мВт – мощность выделяемая всеми компонентами платы;

Sn = 80 мм2 – площадь платы;

Т0С = 40С – максимальная температура окружающей среды;

T = 85С = максимально допустимая температура транзистора.

Tнэ=(2.6∙10-3∙533)/0.49+(24.82∙10-3∙533)/80+40=42.99С

Следовательно заданный тепловой режим соблюдается.

Температура остальных транзисторов рассчитывается аналогично с помощью программы MathCad. Результаты расчётов представлены в Таблице10.2

Таблица 10.2

Из таблицы видно, что для всех транзисторов заданный тепловой режим соблюдается. Следовательно и тепловые условия для всей схемы выполняются.

studfiles.net

Тепловая мощность электрического тока и ее практическое применение

Причина нагревания проводника кроется в том, что энергия движущихся в нем электронов (иными словами, энергия тока) при последовательном столкновении частиц с ионами молекулярной решётки металлического элемента преобразуется в тёплый тип энергии, или Q, так образуется понятие «тепловая мощность».

Работу тока измеряют с помощью международной системы единиц СИ, применяя к ней джоули (Дж), мощность тока определяют как «ватт» (Вт). Отступая от системы на практике, могут применять в том числе и внесистемные единицы, измеряющие работу тока. Среди них ватт-час (Вт × ч), киловатт-час (сокращённо кВт × ч). Например, 1 Вт × ч обозначает работу тока с удельной мощностью 1 ватт и длительностью времени на один час.

Если электроны движутся по неподвижному проводнику из металла, в этом случае вся полезная работа вырабатываемого тока распределяется на нагревание металлической конструкции, и, исходя из положений закона сохранения энергии, это можно описать формулой Q=A=IUt=I2Rt=(U2/R)*t. Такие соотношения с точностью выражают известный закон Джоуля-Ленца. Исторически он впервые был определён опытным путём учёным Д. Джоулем в середине 19-го века, и в то же время независимо от него ещё одним учёным — Э.Ленцем. Практическое применение тепловая мощность нашла в техническом исполнении с изобретения в 1873 году русским инженером А. Ладыгиным обыкновенной лампы накаливании.

Тепловая мощность тока задействуется в целом ряде электрических приборов и промышленных установок, а именно, в тепловых измерительных приборах, нагревательного типа электрических печках, электросварочной и инвенторной аппаратуре, очень распространены бытовые приборы на электрическом нагревательном эффекте – кипятильники, паяльники, чайники, утюги.

Находит себя тепловой эффект и в пищевой промышленности. С высокой долей использования применяется возможность электроконтактного нагрева, что гарантирует тепловая мощность. Он обуславливается тем, что ток и его тепловая мощность, оказывая влияние на пищевой продукт, который обладает определённой степенью сопротивления, вызывает в нем равномерное разогревание. Можно привести в пример то, как производятся колбасные изделия: через специальный дозатор мясной фарш поступает в металлические формы, стенки которых одновременно служат электродами. Здесь обеспечивается постоянная равномерность нагрева по всей площади и объёму продукта, поддерживается заданная температура, сохраняется оптимальная биологическая ценность пищевого продукта, вместе с этими факторами длительность технологических работ и расход энергии остаются наименьшими.

Удельная тепловая мощность электрического тока (ω), иными словами — количество теплоты, что выделяется в единице объёма за определённую единицу времени, рассчитывается следующим образом. Элементарный цилиндрический объём проводника (dV), с поперечным проводниковым сечением dS, длиной dl, параллельной направлению тока, и сопротивлением составляют уравнения R=p(dl/dS), dV=dSdl.

Согласно определениям закона Джоуля-Ленца, за отведённое время (dt) во взятом нами объёме выделится уровень теплоты, равный dQ=I2Rdt=p(dl/dS)(jdS)2dt=pj2dVdt. В таком случае ω=(dQ)/(dVdt)=pj2 и, применяя здесь закон Ома для установления плотности тока j=γE и соотношение p=1/γ, мы сразу получаем выражение ω=jE= γE2. Оно в дифференциальной форме даёт понятие о законе Джоуля-Ленца.

fb.ru

Страничка эмбеддера » Тепловые расчеты

Все электронные компоненты выделяют тепло, поэтому умение рассчитывать радиаторы так, чтобы не пролетать в прикидках на пару порядков очень полезно любому электронщику.

Тепловые расчеты очень просты и имеют очень много общего с расчетами электронных схем. Вот, посмотрите на обычную задачу теплового расчета, с которой я только что столкнулся

Задача

Нужно выбрать радиатор для 5-вольтового линейного стабилизатора, который питается от 12вольт максимум и выдает 0.5А. Максимальная выделяемая мощность получается (12-5)*0.5 = 3.5Вт

Погружение в теорию

Для того, чтобы не плодить сущностей, люди почесали тыковку и поняли, что тепло очень похоже на электрической ток, и для тепловых расчетов можно использовать обычный закон Ома, только

    Напряжение (U) заменяется температурой (T)

    Ток (I) заменяется мощностью (P)

    Сопротивление заменяется тепловым сопротивлением. Обычное сопротивление имеет размерность Вольт/Ампер, а тепловое – °C/Ватт

В итоге, закон Ома заменяется на свой тепловой аналог:

Небольшой замечание – для того, чтобы обозначить, что имеется ввиду тепловое (а не электрическое) сопротивление, к букве R, дописывают букву тэта:на клавиатуре у меня такой буквы нет, а копировать из таблицы символов лень, поэтому я буду пользоваться просто буквой R.

Продолжаем

Тепло выделяется в кристалле стабилизатора, а наша цель – не допустить его перегрева (не допустить перегрева именно кристалла, а не корпуса, это важно!).

До какой температуры можно нагревать кристалл, написано в даташите:

Обычно, предельную температуру кристалла называют Tj (j = junction = переход – термочувствительные внутренности микросхем в основном состоят из pn переходов. Можно считать, что температура переходов равна температуре кристалла)

Без радиатора

Тепловая схема выглядит очень просто:

Специально для случаев использования корпуса без радиатора, в даташитах пишут тепловое сопротивление кристалл-атмосфера (Rj-a) (что такое j вы уже в курсе, a = ambient = окружающая среда)

Заметьте, что температура “земли” не нулевая, а равняется температуре окружающего воздуха (Ta). Температура воздуха зависит от того, в каких условиях находится радиатор Если стоит на открытом воздухе, то можно положить Ta = 40 °C, а вот, если в закрытой коробке, то температура может быть значительно выше!

Записываем тепловой закон Ома: Tj = P*Rj-a + Ta. Подставляем P = 3.5, Rj-a = 65, получаем Tj = 227.5 + 40 = 267.5 °C. Многовато, однако!

Цепляем радиатор

Тепловая схема нашего примера со стабилизатором на радиаторе становится вот такой:

  • Rj-c – сопротивление от кристалла до теплоотвода корпуса (c = case = корпус). Дается в даташите. В нашем случае – 5 °C/Вт – из даташита
  • Rc-r – сопротивление корпус-радиатор. Тут не все так просто. Это сопротивление зависит от того, что находится между корпусом и радиатором. К примеру, силиконовая прокладка имеет коэффициент теплопроводности 1-2 Вт/(м*°C), а паста КПТ-8 – 0.75Вт/(м*°C). Тепловое сопротивление можно получить из коэффициента теплопроводности по формуле:

    R = толщина прокладки/(коэффициент теплопроводности * площадь одной стороны прокладки)

    Часто Rc-r вообще можно игнорировать. К примеру, в нашем случае (используем корпус TO220, с пастой КПТ-8, средняя глубина пасты, взятая с потолка – 0.05мм). Итого, Rc-r = 0.5 °C/Вт. При мощности 3.5вт, разница температур корпуса стабилизатора и радиатора — 1.75градуса. Это – не много. Для нашего примера, возьмем Rc-r = 2 °C/Вт

  • Rr-a – тепловое сопротивление между радиатором и атмосферой. Определяется геометрией радиатора, наличием обдува, и кучей других факторов. Этот параметр намного проще измерить, чем посчитать (см в конце статьи). Для примера — Rr-c = 12.5 °C/Вт

    Ta = 40°C – тут мы прикинули, что атмосферная температура редко выше, можно взять и 50 градусов, чтобы уж точно было.

Подставляем все эти данные в закон Ома, и получаем Tj = 3.5*(5+2+12.5) + 40 = 108.25 °C

Это значительно меньше, чем предельные 150 °C. Такой радиатор можно использовать. При этом, корпус радиатора будет греться до Tc = 3.5*12.5 + 40 = 83.75 °C. Такая температура уже способна размягчить некоторые пластики, поэтому нужно быть осторожным.

Измерение сопротивления радиатор-атмосфера.

Скорее-всего, у вас уже валяется куча радиаторов, которые можно задействовать. Тепловое сопротивление измеряется очень легко. Это этого нужно сопротивление и источник питания.

Лепим сопротивление на радиатор, используя термопасту:

Подключаем источник питания, и выставляем напряжение так, чтобы на сопротивлении выделялась некая мощность. Лучше, конечно, нагревать радиатор той мощностью, которую он будет рассеивать в конечном устройстве (и в том положении, в котором он будет находиться, это важно!). Я обычно оставляю такую конструкцию на пол часа, чтобы она хорошо прогрелась.

После того, как измерили температуру, можно рассчитать тепловое сопротивление

Rr-a = (T-Ta)/P. К примеру, у меня радиатор нагрелся до 81 градуса, а температура воздуха – 31 градус. таким образом, Rr-a = 50/4 = 12.5 °C/Вт.

Прикидка площади радиатора

В древнем справочнике радиолюбителя приводился график, по которому можно прикинуть площадь радиатора. Вот он:

Работать с ним очень просто. Выбираем перегрев, который хочется получить и смотрим, какая площадь соответствует необходимой мощности при таком перегреве.

К примеру, при мощности 4вт и перегреве 20 градусов, понадобится 250см^2 радиатора. Этот график дает завышенную оценку площади, и не учитывает кучу факторов как то принудительный обдув, геометрия ребер, итп.

bsvi.ru


Сообщение от администратора:

Ребята! Кто давно хотел выучить английский?
Переходите по и получите два бесплатных урока в школе английского языка SkyEng!
Занимаюсь там сам — очень круто. Прогресс налицо.

В приложении можно учить слова, тренировать аудирование и произношение.

Попробуйте. Два урока бесплатно по моей ссылке!
Жмите

Количество теплоты, выделяемое в единицу времени в рассматриваемом участке цепи, пропорционально произведению квадрата силы тока на этом участке и сопротивлению участка

Закон Джоуля Ленца в интегральной форме в тонких проводах:

Если сила тока изменяется со временем, проводник неподвижен и химических превращений в нем нет, то в проводнике выделяется тепло.

— Мощность тепла, выделяемого в единице объёма среды при протекании электрического тока, пропорциональна произведению плотности электрического тока на величину электрического поля

Преобразование электрической энергии в тепловую широко используется в электрических печах и различных электронагревательных приборах. Тот же эффект в электрических машинах и аппаратах приводит к непроизвольным затратам энергии (потере энергии и снижении КПД). Тепло, вызывая нагрев этих устройств, ограничивает их нагрузку; при перегрузке повышение температуры может вызвать повреждение изоляции или сокращение срока службы установки.

В формуле мы использовали:

Количество теплоты

Работа тока

Напряжение в проводнике

Сила тока в проводнике

Промежуток времени

Джеймс Прескотт Джоуль (слева) и Эмилий Христианович Ленц (справа)

Электрические нагреватели всевозможных типов используются человечеством уже столетия, благодаря свойству электрического тока выделять тепло при прохождении через проводник. У этого явления есть и негативный фактор – перегретая электропроводка из-за слишком большого тока часто становилась причиной короткого замыкания и возникновения пожаров. Выделение тепла от работы электрического тока изучалось в школьном курсе физики, но многие позабыли эти знания.

Впервые зависимость выделения теплоты от силы электрического тока была сформулирована и математически определена Джеймсом Джоулем в 1841 году, и чуть позже, в 1842 г., независимо от него, Эмилем Ленцем. В честь этих физиков и был назван закон Джоуля-Ленца, по которому рассчитывают мощность электронагревателей и потери на тепловыделение в линиях электропередач.

Определение закона Джоуля – Ленца

В словесном определении, согласно исследований Джоуля и Ленца закон звучит так:

Количество теплоты, выделяемой в определенном объеме проводника при протекании электрического тока прямо пропорционально умножению плотности электрического тока и величины напряженности электрического поля

В виде формулы данный закон выглядит следующим образом:


Выражение закона Джоуля — Ленца

Поскольку описанные выше параметры редко применяются в обыденной жизни, и, учитывая, что почти все бытовые расчеты выделения теплоты от работы электрического тока касаются тонких проводников (кабели, провода, нити накаливания, шнуры питания, токопроводящие дорожки на плате и т. п.), используют закон Джоуля Ленца с формулой, представленной в интегральном виде:


Интегральная форма закона

В словесном определении закон Джоуля Ленца звучит так:


Словесное определение закона Джоуля — Ленца

Если принять, что сила тока и сопротивление проводника не меняется в течение времени, то закон Джоуля — Ленца можно записать в упрощенном виде:

Применив закон Ома и алгебраические преобразования, получаем приведенные ниже эквивалентные формулы:


Эквивалентные выражения теплоты согласно закона Ома

Применение и практическое значение закона Джоуля – Ленца

Исследования Джоуля и Ленца в области тепловыделения от работы электрического тока существенно продвинули научное понимание физических процессов, а выведенные основные формулы не претерпели изменений и используются по сей день в различных отраслях науки и техники. В сфере электротехники можно выделить несколько технических задач, где количество выделяемой при протекании тока теплоты имеет критически важное значение при расчете таких параметров:

  • теплопотери в линиях электропередач;
  • характеристики проводов сетей электропроводки;
  • тепловая мощность (количество теплоты) электронагревателей;
  • температура срабатывания автоматических выключателей;
  • температура плавления плавких предохранителей;
  • тепловыделение различных электротехнических аппаратов и элементов радиотехники.

Электроприборы, в которых используется тепловая работа тока

Тепловое действие электрического тока в проводах линий электропередач (ЛЭП) является нежелательным из-за существенных потерь электроэнергии на тепловыделение.

По различным данным в линиях электропередач теряется до 40% всей производимой электрической энергии в мире. Для уменьшения потерь при передаче электроэнергии на большие расстояния, поднимают напряжение в ЛЭП, производя расчеты по производным формулам закона Джоуля – Ленца.


Диаграмма всевозможных потерь электроэнергии, среди которых теплопотери на воздушных линиях составляют львиную долю (64%)

Очень упрощенно тепловую работу тока можно описать следующим образом: двигаются электроны между молекулами, и время от времени сталкиваются с ними, отчего их тепловые колебания становятся более интенсивными. Наглядная демонстрация тепловой работы тока и ассоциативные пояснения процессов показаны на видео ниже:

Расчеты потерь электроэнергии в линиях электропередач

В качестве примера можно взять гипотетический участок линии электропередач от электростанции до трансформаторной подстанции. Поскольку провода ЛЭП и потребитель электроэнергии (трансформаторная подстанция) соединены последовательно , то через них течет один и тот же ток I. Согласно рассматриваемому тут закону Джоуля – Ленца количество выделяемой на проводах теплоты Q w (теплопотерь) рассчитывается по формуле:

Производимая электрическим током мощность (Q c) в нагрузке рассчитывается согласно закону Ома:

Таким образом, при равенстве токов, в первую формулу можно вставить вместо I выражение Q c /U c , поскольку I = Q c /U c:

Если проигнорировать зависимость сопротивления проводников от изменения температуры, то можно считать R w неизменным (константой). Таким образом, при стабильном энергопотреблении потребителя (трансформаторной подстанции), тепловыделение в проводах ЛЭП будет обратно пропорционально квадрату напряжения в конечной точке линии. Другими словами, чем больше напряжение электропередачи, тем меньше потери электроэнергии.


Для передачи электроэнергии высокого напряжения требуются большие опоры ЛЭП

Работа закона Джоуля – Ленца в быту

Данные расчеты справедливы также и в быту при передаче электроэнергии на малые расстояния – например, от ветрогенератора до инвертора. При автономном энергоснабжении ценится каждый Ватт выработанной низковольтным ветряком энергии, и возможно, будет выгодней поднять напряжение трансформатором прямо у ветрогенератора, чем тратиться на большое сечение кабеля, чтобы уменьшить потери электроэнергии при передаче.


При значительном удалении низковольтного ветрогенератора переменного тока для уменьшения потерь электроэнергии будет выгодней подключение через повышающий трансформатор

В бытовых сетях электропроводки расстояния крайне малы, чтобы уменьшения тепловых потерь поднимать напряжение, поэтому при расчете проводки учитывается тепловая работа тока, согласно закону Джоуля – Ленца при выборе поперечного сечения проводов, чтобы их тепловой нагрев не привел к оплавлению и возгоранию изоляции и окружающих материалов. Выбор кабеля по мощности и электропроводки проводятся согласно таблиц и нормативных документов ПУЭ, и подробно описаны на других страницах данного ресурса.


Соотношения силы тока и поперечного сечения проводников

При расчете температуры нагрева радиотехнических элементов, биметаллической пластины автоматического выключателя или плавкого предохранителя используется закон Джоуля – Ленца в интегральной форме, так как при росте температуры изменяется сопротивление данных материалов. При данных сложных расчетах также учитываются теплоотдача, нагрев от других источников тепла, собственная теплоемкость и множество других факторов.


Программное моделирование тепловыделения полупроводникового прибора

Полезная тепловая работа электрического тока

Тепловыделяющая работа электрического тока широко применяется в электронагревателях, в которых используется последовательное соединение проводников с различным сопротивлением. Данный принцип работает следующим образом: в соединенных последовательно проводниках течет одинаковый ток, значит, согласно закону Джоуля – Ленца, тепла выделится больше у материала проводника с большим сопротивлением.


Спираль с повышенным сопротивлением накаляется, но питающие провода остаются холодными

Таким образом, шнур питания и подводящие провода электроплитки остаются относительно холодными, в то время как нагревательный элемент нагревается до температуры красного свечения. В качестве материала для проводников нагревательных элементов используются сплавы с повышенным (относительно меди и алюминия электропроводки) удельным сопротивлением — нихром, константан, вольфрам и другие.


Нить лампы накаливания изготовляют из тугоплавких вольфрамовых сплавов

При параллельном соединении проводников тепловыделение будет больше на нагревательном элементе с меньшим сопротивлением, так как при его уменьшении возрастает ток относительного соседнего компонента цепи. В качестве примера можно привести очевидный пример свечения двух лампочек накаливания различной мощности – у более мощной лампы тепловыделение и световой поток больше.

Если прозвонить омметром лампочки, то окажется, что у более мощной лампы сопротивление меньше. На видео ниже автор демонстрирует последовательное и параллельное подключение, но к сожалению, он ошибся в комментарии — будет ярче светить лампа с большим сопротивлением, а не наоборот.

Закон Джоуля – Ленца – закон физики, определяющий количественную меру теплового действия электрического тока. Сформулирован этот закон был в 1841 году английским учёным Д. Джоулем и совершенно отдельно от него в 1842 году известным русским физиком Э. Ленцем. Поэтому он получил своё двойное название — закон Джоуля – Ленца.

Определение закона и формула

Словесная формулировка имеет следующий вид: мощность тепла, выделяемого в проводнике при протекании сквозь него , пропорционально произведению значения плотности электрического поля на значение напряженности.

Математически закон Джоуля — Ленца выражается следующим образом:

ω = j E = ϭ E²,

где ω — количество тепла, выделяемого в ед. объема;

E и j – напряжённость и плотность, соответственно, электрического полей;

σ — проводимость среды.

Физический смысл закона Джоуля – Ленца

Закон можно объяснить следующим образом: ток, протекая по проводнику, представляет собой перемещение электрического заряда под воздействием . Таким образом, электрическое поле совершает некоторую работу. Эта работа расходуется на нагрев проводника.

Другими словами, энергия переходит в другое свое качество – тепло.

Но чрезмерный нагрев проводников с током и электрооборудования допускать нельзя, поскольку это может привести к их повреждению. Опасен сильный перегрев при проводов, когда по проводниках могут протекать достаточно большие токи.

В интегральной форме для тонких проводников закон Джоуля – Ленца звучит следующим образом: количество теплоты, которое выделяется в единицу времени в рассматриваемом участке цепи, определяется как произведение квадрата силы тока на сопротивление участка.

Математически эта формулировка выражается следующим образом:

Q = ∫ k I² R t,

при этом Q – количество выделившейся теплоты;

I – величина тока;

R — активное сопротивление проводников;

t – время воздействия.

Значение параметра k принято называть тепловым эквивалентом работы. Величина этого параметра определяется в зависимости от разрядности единиц, в которых выполняются измерения значений, используемых в формуле.

Закон Джоуля-Ленца имеет достаточно общий характер, поскольку не имеет зависимости от природы сил, генерирующих ток.

Из практики можно утверждать, что он справедлив, как для электролитов, так проводников и полупроводников.

Область применения

Областей применения в быту закона Джоуля Ленца – огромное количество. К примеру, вольфрамовая нить в лампе накаливания, дуга в электросварке, нагревательная нить в электрообогревателе и мн. др. Это наиболее широко распространенный физический закон в повседневной жизни.

НАГРЕВАТЕЛЬНЫЙ ЭФФЕКТ ЭЛЕКТРИЧЕСКОГО ТОКА

Введение

Когда ток течет по проводнику, в проводнике генерируется тепловая энергия. Нагревательный эффект электрического тока зависит от трех факторов:

  • Сопротивление R проводника. Чем выше сопротивление, тем больше тепла.
  • Время t, в течение которого течет ток. Чем больше время, тем больше выделяется тепла.
  • Величина тока I. Чем выше сила тока, тем больше выделяется тепла.

Следовательно, эффект нагрева, создаваемый электрическим током I через проводник сопротивления R в течение некоторого времени, t определяется как H = I 2 Rt. Это уравнение называется уравнением Джоуля электрического нагрева.

Электроэнергия и мощность

Работа, выполняемая при проталкивании заряда по электрической цепи, определяется выражением w.d = VIt

.

Так что мощность, P = w.d / t = VI

Электрическая мощность, потребляемая электроприбором, определяется как P = VI = I 2 R = V 2 / R

Пример

  1. Электрическая лампочка имеет маркировку 100 Вт, 240 В.Вычислить:
а) Ток через нить накала при нормальной работе лампы
б) Сопротивление нити накала лампы.

Решение

  1. I = P / V = ​​100/240 = 0,4167A
  2. R = P / I 2 = 100 / 0,4167 2 = 576,04 Ом или R = V 2 / P = 240 2 /100 = 576 Ом
  1. Найдите энергию, рассеиваемую за 5 минут электрической лампочкой с нитью накала 500 Ом, подключенной к источнику питания 240 В.{ ANS. 34,560J }

Решение

E = Pt = V2 / R * t = (240 2 * 5 * 60) / 500 = 34,560 Дж

  1. Для нагрева воды используется погружной нагреватель мощностью 2,5 кВт. Вычислить:
  1. Рабочее напряжение нагревателя при его сопротивлении 24 Ом
  2. Электрическая энергия, преобразованная в тепловую за 2 часа.

{ ANS. 244,9488 В, 1,8 * 10 7 Дж }

Решение

  1. P = VI = I 2 R

I = (2500/24) 1/2 = 10.2062A

В = ИК = 10,2062 * 24 = 244,9488 В

  1. E = VIt = Pt = 2500 * 2 * 60 * 60 = 1,8 * 10 7 J

ИЛИ E = VIt = 244,9488 * 10,2062 * 2 * 60 * 60 = 1,8 * 10 7 Дж

Электрическая лампочка имеет маркировку 100W, 240V. Вычислить:
Ток через нить накала
Сопротивление нити накала лампы.

Решение

P = VI I = P / V = ​​100/240 = 0,4167A
Согласно закону Ома, V = IR R = V / I = 240 / 0,4167 = 575,95 Ом

Применение нагревающего эффекта электрического тока

Большинство бытовых электроприборов таким образом преобразуют электрическую энергию в тепло.К ним относятся лампы накаливания, электрический нагреватель, электрический утюг, электрический чайник и т. Д.

В осветительных приборах

  1. Лампы накаливания — изготовлены из вольфрамовой проволоки, заключенной в стеклянную колбу, из которой удален воздух. Это потому, что воздух окисляет нить. Нить нагревается до высокой температуры и становится раскаленной добела. Вольфрам используется из-за его высокой температуры плавления; 3400 0 Колба заполнена неактивным газом, например. аргон или азот при низком давлении, что снижает испарение вольфрамовой проволоки.Однако одним из недостатков инертного газа является то, что он вызывает конвекционные токи, которые охлаждают нить накала. Эта проблема сводится к минимуму за счет наматывания проволоки таким образом, чтобы она занимала меньшую площадь, что снижает потери тепла за счет конвекции.
  2. Люминесцентные лампы — эти лампы более эффективны по сравнению с лампами накаливания и служат намного дольше. У них есть пары ртути в стеклянной трубке, которая при включении испускает ультрафиолетовое излучение. Это излучение заставляет порошок в трубке светиться (флуоресцировать) i.е. излучает видимый свет. Из разных порошков получаются разные цвета. Обратите внимание, что люминесцентные лампы дороги в установке, но их эксплуатационные расходы намного меньше.

В электрическом обогреве

  1. Электрические плиты — электрические плиты раскалены докрасна, и произведенная тепловая энергия поглощается кастрюлей за счет теплопроводности.
  2. Электрические обогреватели — лучистые обогреватели становятся красными при температуре около 900 0 C, а испускаемое излучение направляется в комнату с помощью полированных отражателей.
  3. Электрочайники — нагревательный элемент размещается на дне чайника так, чтобы нагреваемая жидкость покрывала его. Затем тепло поглощается водой и распределяется по всей жидкости за счет конвекции.
  4. Электрические утюги — при прохождении тока через нагревательный элемент выделяемая тепловая энергия передается на основание из тяжелого металла, повышая его температуру. Затем эта энергия используется для прессования одежды. Температуру утюга можно контролировать с помощью термостата (биметаллической планки).

Проводимость | Физика

Цели обучения

К концу этого раздела вы сможете:

  • Рассчитайте теплопроводность.
  • Наблюдать за теплопроводностью при столкновении.
  • Изучение теплопроводности обычных веществ.

Рис. 1. Изоляция используется для ограничения теплопроводности изнутри наружу (зимой) и снаружи внутрь (летом).(кредит: Джайлз Дуглас)

Вам холодно в ногах, когда вы идете босиком по ковру в гостиной в холодном доме, а затем ступаете на плиточный пол кухни. Этот результат интригует, так как ковер и кафельный пол имеют одинаковую температуру. Различные ощущения, которые вы испытываете, объясняются разной скоростью теплопередачи: потери тепла в течение одного и того же промежутка времени больше для кожи, контактирующей с плиткой, чем с ковром, поэтому перепад температуры больше на плитке.

Некоторые материалы проводят тепловую энергию быстрее, чем другие. В целом, хорошие проводники электричества (металлы, такие как медь, алюминий, золото и серебро) также являются хорошими проводниками тепла, тогда как изоляторы электричества (дерево, пластик и резина) являются плохими проводниками тепла. На рисунке 2 показаны молекулы в двух телах при разных температурах. (Средняя) кинетическая энергия молекулы в горячем теле выше, чем в более холодном теле. Если две молекулы сталкиваются, происходит передача энергии от горячей молекулы к холодной.Кумулятивный эффект от всех столкновений приводит к чистому потоку тепла от горячего тела к более холодному телу. Таким образом, тепловой поток зависит от разности температур Δ Τ = Τ горячий T холодный . Таким образом, вы получите более сильный ожог от кипятка, чем от горячей воды из-под крана. И наоборот, если температуры одинаковы, чистая скорость теплопередачи падает до нуля и достигается равновесие. Благодаря тому, что количество столкновений увеличивается с увеличением площади, теплопроводность зависит от площади поперечного сечения.Если прикоснуться ладонью к холодной стене, рука остынет быстрее, чем при прикосновении к ней кончиком пальца.

Рис. 2. Молекулы в двух телах при разных температурах имеют разные средние кинетические энергии. Столкновения, происходящие на контактной поверхности, имеют тенденцию передавать энергию из высокотемпературных областей в низкотемпературные области. На этом рисунке молекула в области более низких температур (правая сторона) имеет низкую энергию перед столкновением, но ее энергия увеличивается после столкновения с контактной поверхностью.Напротив, молекула в области более высоких температур (слева) имеет высокую энергию до столкновения, но ее энергия уменьшается после столкновения с контактной поверхностью.

Третий фактор в механизме теплопроводности — это толщина материала, через который передается тепло. На рисунке ниже показана плита из материала с разными температурами с обеих сторон. Предположим, что T 2 больше, чем T 1 , так что тепло передается слева направо.Передача тепла с левой стороны на правую осуществляется серией столкновений молекул. Чем толще материал, тем больше времени требуется для передачи того же количества тепла. Эта модель объясняет, почему толстая одежда зимой теплее, чем тонкая, и почему арктические млекопитающие защищаются толстым салом.

Рис. 3. Теплопроводность происходит через любой материал, представленный здесь прямоугольной полосой, будь то оконное стекло или моржовый жир. Температура материала составляет T 2 слева и T 1 справа, где T 2 больше, чем T 1 .Скорость теплопередачи прямо пропорциональна площади поверхности A, разности температур T 2 T 1 и проводимости вещества k . Скорость теплопередачи обратно пропорциональна толщине d .

Наконец, скорость теплопередачи зависит от свойств материала, описываемых коэффициентом теплопроводности. Все четыре фактора включены в простое уравнение, выведенное из экспериментов и подтвержденное экспериментами.Скорость кондуктивной теплопередачи через пластину материала, такую ​​как показанная на рисунке 3, равна

.

[латекс] \ displaystyle \ frac {Q} {t} = \ frac {kA \ left (T_2-T_1 \ right)} {d} \\ [/ latex],

, где [латекс] \ frac {Q} {t} \\ [/ latex] — скорость теплопередачи в ваттах или килокалориях в секунду, k — теплопроводность материала, A и d — это его площадь поверхности и толщина, как показано на Рисунке 3, а ( T 2 T 1 ) — разность температур на плите.В таблице 1 приведены типичные значения теплопроводности.

Пример 1. Расчет теплопроводности: скорость теплопроводности через ледяной ящик

Ледяной ящик из пенополистирола имеет общую площадь 0,950 м 2 и стенки со средней толщиной 2,50 см. В коробке есть лед, вода и напитки в банках с температурой 0 ° C. Внутренняя часть ящика охлаждается за счет таяния льда. Сколько льда тает за сутки, если хранить ледяной ящик в багажнике автомобиля при температуре 35,0ºC?

Стратегия

Этот вопрос включает как тепло для фазового перехода (таяние льда), так и передачу тепла за счет теплопроводности.{\ circ} \ text {C}; \\ t & = & 1 \ text {day} = 24 \ text {hours} = 86 400 \ text {s}. \ end {array} \\ [/ latex]

Определите неизвестные. Нам нужно найти массу льда м . Нам также нужно будет вычислить чистое тепло, передаваемое для таяния льда, Q . Определите, какие уравнения использовать. Скорость теплопередачи за счет теплопроводности определяется как

.

[латекс] \ displaystyle \ frac {Q} {t} = \ frac {kA \ left (T_2-T_1 \ right)} {d} \\ [/ latex]

Тепло используется для плавления льда: Q мл f .{\ circ} \ text {C} \ right)} {0,0250 \ text {m}} = 13,3 \ text {J / s} \\ [/ latex]

Умножьте скорость теплопередачи на время (1 день = 86 400 с): Q = [латекс] \ left (\ frac {Q} {t} \ right) t \\ [/ latex] = ( 13,3 Дж / с) (86400 с) = 1,15 × 10 6 Дж

Установите равным теплу, передаваемому для растапливания льда: Q = мл f . Решить относительно массы m :

[латекс] \ displaystyle {m} = \ frac {Q} {L _ {\ text {f}}} = \ frac {1.3 \ text {Дж / кг}} = 3,44 \ text {кг} \\ [/ latex]

Обсуждение

Результат 3,44 кг, или около 7,6 фунта, кажется примерно правильным, если судить по опыту. Вы можете рассчитывать использовать мешок льда весом около 4 кг (7–10 фунтов) в день. Если вы добавляете горячую пищу или напитки, потребуется немного льда.

Проверка проводимости в таблице 1 показывает, что пенополистирол — очень плохой проводник и, следовательно, хороший изолятор. Среди других хороших изоляторов — стекловолокно, шерсть и перья из гусиного пуха. Как и пенополистирол, все они включают в себя множество маленьких карманов с воздухом, благодаря низкой теплопроводности воздуха.

Таблица 1. Теплопроводность обычных веществ
Вещество Теплопроводность, k (Дж / с⋅м⋅ºC)
Серебро 420
Медь 390
Золото 318
Алюминий 220
Стальной чугун 80
Сталь (нержавеющая) 14
Лед 2.2
Стекло (среднее) 0,84
Бетонный кирпич 0,84
Вода 0,6
Жировая ткань (без крови) 0,2 ​​
Асбест 0,16
Гипсокартон 0,16
Дерево 0,08–0,16
Снег (сухой) 0,10
Пробка 0.042
Стекловата 0,042
Шерсть 0,04
Пуховые перья 0,025
Воздух 0,023
Пенополистирол 0,010

Рис. 4. Стекловолокно используется для изоляции стен и потолков, чтобы предотвратить теплопередачу между внутренней частью здания и внешней средой.

Комбинацией материала и толщины часто манипулируют для создания хороших изоляторов — чем меньше проводимость k и чем больше толщина d , тем лучше.Соотношение [латекс] \ frac {d} {k} \\ [/ latex], таким образом, будет большим для хорошего изолятора. Отношение [латекс] \ frac {d} {k} \\ [/ latex] называется коэффициентом R . Скорость кондуктивной теплопередачи обратно пропорциональна R . Чем больше значение R , тем лучше изоляция. R Коэффициент чаще всего указывается для бытовой теплоизоляции, холодильников и т.п. — к сожалению, он все еще выражается в неметрических единицах футов 2 · ° F · ч / британских тепловых единиц, хотя единицы обычно не указываются (1 британский тепловая единица [BTU] — это количество энергии, необходимое для изменения температуры на 1.0 фунтов воды при температуре 1,0 ° F). Пара типичных значений: коэффициент R, , равный 11, для стекловолоконных войлоков (кусков) изоляции толщиной 3,5 дюйма и коэффициент R, равный , равный 19, для стекловолоконных войлоков толщиной, так и 6,5 дюймов. Стены обычно утепляются 3,5-дюймовыми ватными покрытиями, а потолки — 6,5-дюймовыми. В холодном климате для потолков и стен можно использовать более толстые войлоки.

Обратите внимание, что в таблице 1 лучшие теплопроводники — серебро, медь, золото и алюминий — также являются лучшими электрическими проводниками, что опять же связано с плотностью свободных электронов в них.Кухонная утварь обычно изготавливается из хороших проводников.

Пример 2. Расчет разницы температур, поддерживаемой теплопередачей: теплопроводность через алюминиевую сковороду

Вода кипит в алюминиевой кастрюле, поставленной на электрический элемент на плите. Дно кастрюли имеет толщину 0,800 см и диаметр 14,0 см. Кипящая вода испаряется со скоростью 1,00 г / с. Какая разница температур на дне сковороды?

Стратегия

Проводимость через алюминий является здесь основным методом теплопередачи, поэтому мы используем уравнение для скорости теплопередачи и решаем разницу температур .

[латекс] \ displaystyle {T} _2-T_1 = \ frac {Q} {t} \ left (\ frac {d} {kA} \ right) \\ [/ latex]

Решение

Определите известные значения и преобразуйте их в единицы СИ Толщина поддона, d = 0,900 см = 8,0 × 10 −3 м площадь поддона, A = π (0,14 / 2) 2 м 2 = 1,54 × 10 −2 м 2 , а теплопроводность k = 220 Дж / с м ° C.

Рассчитайте необходимую теплоту испарения 1 г воды: Q = мл v = (1.{\ circ} \ text {C} \\ [/ latex]

Обсуждение

Значение теплопередачи [латекс] \ frac {Q} {t} \ [/ latex] = 2,26 кВт или 2256 Дж / с типично для электрической плиты. Это значение дает очень небольшую разницу температур между плитой и сковородой. Учтите, что конфорка печи раскалилась докрасна, а температура внутри сковороды почти 100ºC из-за контакта с кипящей водой. Этот контакт эффективно охлаждает дно сковороды, несмотря на его близость к очень горячей конфорке плиты.Алюминий является настолько хорошим проводником, что достаточно лишь этой небольшой разницы температур для передачи тепла в сковороду 2,26 кВт.

Проводимость вызывается случайным движением атомов и молекул. По сути, это неэффективный механизм переноса тепла на макроскопические расстояния и короткие временные расстояния. Возьмем, к примеру, температуру на Земле, которая была бы невыносимо холодной ночью и чрезвычайно высокой днем, если бы перенос тепла в атмосфере происходил только за счет теплопроводности.В другом примере автомобильные двигатели будут перегреваться, если не будет более эффективного способа отвода избыточного тепла от поршней.

Проверьте свое понимание

Как изменяется скорость теплопередачи за счет теплопроводности, когда все пространственные размеры удваиваются?

Решение

Поскольку площадь является произведением двух пространственных измерений, она увеличивается в четыре раза, когда каждое измерение удваивается ( A final = (2 d ) 2 = 4 d 2 = 4 А начальный ).Однако расстояние просто удваивается. Поскольку разница температур и коэффициент теплопроводности не зависят от пространственных размеров, скорость передачи тепла за счет теплопроводности увеличивается в четыре раза, деленные на два или два:

[латекс] \ left (\ frac {Q} {t} \ right) _ {\ text {final}} = \ frac {kA _ {\ text {final}} \ left (T_2-T_1 \ right)} {d_ {\ text {final}}} = \ frac {k \ left (4A _ {\ text {initial}} \ right) \ left (T_2-T_1 \ right)} {2d _ {\ text {initial}}} = 2 \ frac {kA _ {\ text {initial}} \ left (T_2-T_1 \ right)} {d _ {\ text {initial}}} = 2 \ left (\ frac {Q} {t} \ right) _ {\ text {initial}} \\ [/ latex]

Сводка раздела

  • Теплопроводность — это передача тепла между двумя объектами, находящимися в непосредственном контакте друг с другом.
  • Скорость теплопередачи [латекс] \ frac {Q} {t} \ [/ latex] (энергия в единицу времени) пропорциональна разнице температур T 2 T 1 и площадь контакта A и обратно пропорциональна расстоянию d между объектами: [latex] \ frac {Q} {t} = \ frac {\ text {kA} \ left ({T} _ {2} — {T} _ {1} \ right)} {d} \\ [/ latex].

Концептуальные вопросы

  1. Некоторые электроплиты имеют плоскую керамическую поверхность со скрытыми нагревательными элементами.Кастрюля, поставленная над нагревательным элементом, будет нагрета, при этом безопасно прикасаться к поверхности всего в нескольких сантиметрах от нее. Почему керамика с проводимостью меньше, чем у металла, но больше, чем у хорошего изолятора, является идеальным выбором для плиты?
  2. Свободная белая одежда, закрывающая большую часть тела, идеальна для обитателей пустыни как на жарком солнце, так и в холодные вечера. Объясните, чем выгодна такая одежда и днем, и ночью.

Рисунок 5.Джеллабию носят многие мужчины в Египте. (кредит: Зерида)

Задачи и упражнения

  1. (a) Рассчитайте коэффициент теплопроводности через стены дома толщиной 13,0 см, у которых средняя теплопроводность в два раза выше, чем у стекловаты. Предположим, что нет ни окон, ни дверей. Площадь стен составляет 120 м 2 2 , их внутренняя поверхность имеет температуру 18,0ºC, а внешняя поверхность — 5,00ºC. (b) Сколько комнатных обогревателей мощностью 1 кВт потребуется для уравновешивания теплопередачи за счет теплопроводности?
  2. Скорость передачи тепла из окна в зимний день достаточно высока, чтобы охладить воздух рядом с ним.Чтобы увидеть, насколько быстро окна передают тепло за счет теплопроводности, рассчитайте коэффициент теплопроводности в ваттах через окно размером 3,00 м 2 толщиной 0,635 см (1/4 дюйма), если температура внутренней и внешней поверхностей составляет 5,00 ºC и −10,0ºC соответственно. Такая высокая скорость не будет поддерживаться — внутренняя поверхность остынет и даже может образоваться иней.
  3. Рассчитайте скорость отвода тепла от тела человека, предполагая, что внутренняя температура ядра составляет 37,0 ° C, а температура кожи равна 34.0ºC, толщина тканей в среднем составляет 1,00 см, а площадь поверхности составляет 1,40 м. 2 .
  4. Предположим, вы стоите одной ногой на керамическом полу и одной ногой на шерстяном ковре, соприкасаясь каждой ногой на площади 80,0 см. 2 . И керамика, и ковер имеют толщину 2,00 см и температуру на нижней стороне 10,0 ° C. С какой скоростью должна происходить теплопередача от каждой ступни, чтобы верхняя часть керамики и ковра поддерживала температуру 33,0 ° C?
  5. Человек потребляет 3000 ккал пищи за один день, преобразовывая большую ее часть для поддержания температуры тела.Если он теряет половину этой энергии из-за испарения воды (при дыхании и потоотделении), сколько килограммов воды испаряется?
  6. (a) Огнеходящий бежит по раскаленному углю, не получив ожогов. Рассчитайте тепло, передаваемое теплопроводностью в подошву одной ступни огнехода, учитывая, что нижняя часть ступни представляет собой мозоль толщиной 3,00 мм с проводимостью на нижнем уровне диапазона для древесины и ее плотность составляет 300 кг / м3. 3 . Площадь контакта 25,0 см. 2 , температура углей 700ºC, время контакта 1.00 с. (b) Какое повышение температуры происходит в 25,0 см 3 пораженной ткани? (c) Как вы думаете, какое влияние это окажет на ткань, учитывая, что каллус состоит из мертвых клеток?
  7. (а) Какова скорость теплопроводности через мех толщиной 3 см у крупного животного с площадью поверхности 1,40 м 2 ? Предположим, что температура кожи животного составляет 32,0 ° C, температура воздуха -5,00 ° C и мех имеет такую ​​же теплопроводность, как воздух.(б) Какой прием пищи потребуется животному в течение одного дня, чтобы восполнить эту теплопередачу?
  8. Морж передает энергию путем теплопроводности через свой жир со скоростью 150 Вт при погружении в воду с температурой –1,00 ° C. Внутренняя температура моржа составляет 37,0ºC, а его площадь поверхности составляет 2,00 м. 2 . Какова средняя толщина его подкожного жира, который имеет проводимость жировых тканей без крови?

    Рис. 6. Морж на льду. (Источник: капитан Бадд Кристман, Корпус NOAA)

  9. Сравните коэффициент теплопроводности через 13.Стена толщиной 0 см, имеющая площадь 10,0 м 2 и удвоенную теплопроводность, чем стекловата, со скоростью теплопроводности через окно толщиной 0,750 см и площадью 2,00 м 2 , предполагая одинаковую разницу температур между ними.
  10. Предположим, что человек покрыт с головы до ног шерстяной одеждой средней толщины 2,00 см и передает энергию путем теплопроводности через одежду со скоростью 50,0 Вт. Какова разница температур в одежде, если площадь поверхности равна 1.40 м 2 ?
  11. Некоторые поверхности плит сделаны из гладкой керамики, что облегчает их очистку. Если керамика имеет толщину 0,600 см и теплопроводность происходит через ту же площадь и с той же скоростью, что и в примере 2, какова разница температур в ней? Керамика имеет такую ​​же теплопроводность, как стекло и кирпич.
  12. Один из простых способов сократить расходы на отопление (и охлаждение) — это добавить дополнительную изоляцию на чердаке дома. Предположим, что в доме уже есть 15 см стекловолоконной изоляции на чердаке и на всех внешних поверхностях.Если добавить на чердак еще 8,0 см стеклопластика, то на какой процент упадет стоимость отопления дома? Возьмем одноэтажный дом размером 10 м на 15 м на 3,0 м. Не обращайте внимания на проникновение воздуха и потерю тепла через окна и двери.
  13. (a) Рассчитайте коэффициент теплопроводности через окно с двойным остеклением, которое имеет площадь 1,50 м 2 и состоит из двух стекол толщиной 0,800 см, разделенных воздушным зазором 1,00 см. Температура внутренней поверхности 15.0ºC, а снаружи −10,0ºC. (Подсказка: на двух стеклянных панелях наблюдаются одинаковые перепады температуры. Сначала найдите их, а затем перепад температуры в воздушном зазоре. Эта проблема игнорирует повышенную теплопередачу в воздушном зазоре из-за конвекции.) (B) Рассчитайте скорость теплопроводность через окно толщиной 1,60 см той же площади и с такими же температурами. Сравните свой ответ с ответом на часть (а).
  14. Многие решения принимаются на основе периода окупаемости: времени, которое потребуется за счет экономии, чтобы равняться капитальным затратам на инвестиции.Приемлемые сроки окупаемости зависят от бизнеса или философии. (Для некоторых отраслей период окупаемости составляет всего два года.) Предположим, вы хотите установить дополнительную изоляцию, о которой идет речь в вопросе 12. Если стоимость энергии составляет 1 доллар США за миллион джоулей, а стоимость изоляции составляет 4 доллара США за квадратный метр, тогда рассчитайте простой срок окупаемости. . Возьмем среднее значение Δ T для 120-дневного отопительного сезона равным 15,0 ° C.
  15. Для человеческого тела, какова скорость теплопередачи через ткани тела при следующих условиях: толщина ткани равна 3.00 см, изменение температуры 2,00ºC, а площадь кожи 1,50 м 2 . Как это соотносится со средней скоростью передачи тепла телу в результате потребления энергии около 2400 ккал в день? (Никакие упражнения не включены.)

Глоссарий

R-фактор: отношение толщины материала к проводимости

скорость кондуктивной теплопередачи: скорость теплопередачи от одного материала к другому

теплопроводность: свойство способности материала проводить тепло

Избранные решения проблем и упражнения

1.(а) 1.01 × 10 3 Вт; (б) Один

3. 84.0 Вт

5. 2,59 кг

7. (а) 39,7 Вт; (б) 820 ккал

9. 35 к 1, окно к стене

11. 1,05 × 10 3 К

13. (а) 83 Вт; (b) в 24 раза больше, чем у окна с двойным остеклением.

15. 20,0 Вт, 17,2% от 2400 ккал в день


Формула расхода тепла

Количество тепла, которое передается в единицу времени в некотором материале.

Скорость теплового потока в стержне из материала пропорциональна площади поперечного сечения стержня и разности температур между концами и обратно пропорциональна длине.

Тепловой поток = — (коэффициент теплопередачи) * (площадь тела) * (изменение температуры) / (длина материала)

Уравнение:

Q = -k (А / л) (ΔT)

У нас:

Q: теплопередача в единицу времени

K: теплопроводность

A: площадь излучающего тела

л: длина материала.

ΔT: Разница температур.

Вопросы по формуле теплопередачи:

1) Стена дома шириной 7 м и высотой 6 м сделана из 0.Кирпич толщиной 3 м при k = 0,6 Вт / мК. Температура внутри стены составляет 16 ° C, а снаружи — 6 ° C. Найдите тепловой поток.

Ответ:

Разница температур ΔT = T i — T O = 16 ° C — 6 ° C = 10 ° C = 283 K.

Тепловой поток определяется по формуле:

Q = -k (А / л) (ΔT)

Подставляя значения коэффициента теплопроводности, площади, длины и разницы температур внутри и снаружи,

Q = -0.6 Вт / м · K (7 м * 6 м / 0,3 м) (283 K) =

Q = -840 Вт

2) Для подачи нагретой воды используется медная труба диаметром 20 мм, внешняя поверхность трубы имеет k = 6 Вт / м · К, ее толщина 2 мм. Найдите тепловой поток на трубе при температуре внешней поверхности 80 ° C и температуре окружающей среды 20 ° C.

Ответ:

Разница температур ΔT = T i — T O = 80 ° C — 20 ° C = 60 ° C = 333 K.

Тепловой поток определяется по формуле:

Q = -k (А / л) (ΔT)

Площадь определяется как π (0.02 м) 2 = π 0,0004 = 0,0012 м 2 .

Подставляя значения коэффициента теплопроводности, площади, длины и разницы температур внутри и снаружи,

Q = -6 Вт / м K (0,0012 м 2 / 0,002 м) (333 K) = -1198,8 Вт

Q = -1198 Вт

ток — Как рассчитать превышение температуры в медном проводнике?

Хотя это вопрос семилетней давности, я подумал, что могу внести свой вклад в подход, который, как я обнаружил, вдохновлен некоторыми моментами, упомянутыми в примечании к применению от SIEMENS.2 $$

$$ I_ {max}: \ text {максимальный непрерывный ток,} I_ {op}: \ text {рабочий ток} $$ $$ \ Theta_ {x}: \ text {x temperature,} \ Theta_ {amb}: \ text {ambient,} \ Delta \ Theta_ {max}: \ Theta \ text {rise @} I_ {max} $$

Максимальный продолжительный рабочий ток

Кабели имеют номинальную пропускную способность по току для непрерывной работы. Различная изоляция кабеля допускает различные максимальные рабочие температуры. Их можно рассчитать в соответствии со стандартами МЭК, но мы можем использовать либо наши спецификации кабелей, либо общие, чтобы получить расчетное значение.oC $$ Это на больше максимальной рабочей температуры кабеля с изоляцией из сшитого полиэтилена. Если это изоляция из ПВХ, расчет дает> 87ºC, где изоляция, вероятно, расплавится. ПВХ при температуре выше 60 ° C становится нестабильным.


Сравнение с отклонениями (поправочные коэффициенты)

Если мы сравним использование этой формулы с отклонениями от номиналов, мы увидим определенную согласованность;

В примечаниях к применению указано, что для других температур окружающего воздуха необходимо применять поправочные коэффициенты для максимального тока:

  | Окр. ºC | 35 | 40 | 45 | 50 | 55 | 60 | 65 | 70 | 75 | 80 |
| Фактор | 1.oC $$ 

Следующие расчетные температуры в установившемся режиме следующие:

  | Окр. ºC | 35 | 40 | 45 | 50 | 55 | 60 | 65 | 70 | 75 | 80 |
| Ток | 26,4 | 25,2 | 24,0 | 22,56 | 21,12 | 19,68 | 17,76 | 16,08 | 13,92 | 11,28 |
| ssTemp | 89.45 | 89.61 | 90.00 | 89.76 | 89.85 | 90.26 | 89.64 | 90.20 | 90.14 | 89.94 |
  

Время, необходимое для достижения установившейся температуры

Сколько времени потребуется для достижения этой температуры, можно оценить, учитывая номинальный ток короткого замыкания кабеля.2 \ приблизительно 3,7 \ text {min} $$

\ tau определяет время, необходимое для достижения 63% конечной температуры. Обычно мы оцениваем, что при 5 * \ tau мы находимся примерно на 99% от конечной температуры. 5 * 3,7 мин = 18,5 мин.

$$ \ tau \ text {действительно для достижения любых расчетных условий устойчивого состояния} $$

$$ \ text {Время достижения любой установившейся температуры} \ приблизительно 5 \ cdot \ tau \ приблизительно 18,5 \ text {min} $$

$$ \ Delta \ Theta_ {ss-amb} = \ Theta_ {устойчивое состояние} - \ Theta_ {amb} $$

Если построить график, то это выглядит следующим образом:


приблизительная / расчетная демонстрация

Наш расчетный \ tau был со значениями: Температура окружающей среды 45ºC, рабочая температура = 90ºC.2 = 0,64 $$

, но наша расчетная \ Delta T (повышение температуры) составляет 70 ° C по сравнению с 45 ° C. $$ K _ {\ Delta \ Theta} \ приблизительно \ frac {\ Delta \ Theta_ {op}} {\ Delta \ Theta_ {ref}} = \ frac {70} {45} \ приблизительно 1,5556 $$

, применив их к нашему \ tau следующим образом, мы получим $$ \ tau_ {op} = \ tau_ {ref} \ cdot K _ {\ tau} \ cdot K _ {\ Delta \ Theta} = 3,7 \ cdot 0,64 \ cdot 1,5556 = 3,68 \ leadsto 5 \ tau = 18,4 \ text {min } $$

Обратите внимание, что эти формулы для демонстрации модифицированного \ tau были изобретены «из воздуха», «ощущением», некоторыми «логическими» соображениями.Это может быть совершенно неверно, и если я сделал «сумасшедшее» предположение, пожалуйста, дайте мне знать, чтобы я мог узнать свою ошибку. Когда-нибудь я сделаю несколько измерений, чтобы проверить это.


Ресурсы

Теплопередача, удельная теплоемкость и калориметрия - University Physics Volume 2

Цели обучения

К концу этого раздела вы сможете:

  • Объясните явления, связанные с теплом как формой передачи энергии
  • Решение проблем, связанных с теплопередачей

В предыдущих главах мы видели, что энергия - одно из фундаментальных понятий физики. Тепло - это тип передачи энергии, который вызывается разницей температур и может изменять температуру объекта. Как мы узнали ранее в этой главе, теплопередача - это движение энергии от одного места или материала к другому в результате разницы температур. Теплопередача является основой таких повседневных действий, как отопление и приготовление пищи, а также многих производственных процессов. Он также составляет основу тем, которые будут рассмотрены в оставшейся части этой главы.

Мы также вводим понятие внутренней энергии, которая может быть увеличена или уменьшена за счет теплопередачи.Мы обсуждаем другой способ изменить внутреннюю энергию системы, а именно выполнение работы над ней. Таким образом, мы начинаем изучение взаимосвязи тепла и работы, которая является основой двигателей и холодильников и центральной темой (и источником названия) термодинамики.

Внутренняя энергия и тепло

Тепловая система имеет внутренней энергии (также называемой тепловой энергией ) , которая является суммой механических энергий ее молекул. Внутренняя энергия системы пропорциональна ее температуре.Как мы видели ранее в этой главе, если два объекта с разной температурой приводят в контакт друг с другом, энергия передается от более горячего объекта к более холодному, пока тела не достигнут теплового равновесия (то есть они имеют одинаковую температуру). Ни один из объектов не совершает никакой работы, потому что никакая сила не действует на расстоянии (как мы обсуждали в разделе Работа и кинетическая энергия). Эти наблюдения показывают, что тепло - это энергия, спонтанно передаваемая из-за разницы температур. (Рисунок) показывает пример теплопередачи.

(а) Здесь безалкогольный напиток имеет более высокую температуру, чем лед, поэтому они не находятся в тепловом равновесии. (b) Когда безалкогольный напиток и лед могут взаимодействовать, тепло передается от напитка ко льду из-за разницы температур, пока они не достигнут одинаковой температуры, что приводит к достижению равновесия. Фактически, поскольку безалкогольный напиток и лед находятся в контакте с окружающим воздухом и скамейкой, конечная равновесная температура будет такой же, как и температура окружающей среды.

Значение «тепла» в физике отличается от его обычного значения.Например, в разговоре мы можем сказать «жара была невыносимой», но в физике мы бы сказали, что температура была высокой. Тепло - это форма потока энергии, а температура - нет. Между прочим, люди чувствительны к тепловому потоку , а не к температуре.

Поскольку тепло - это форма энергии, в системе СИ единицей измерения является джоуль (Дж). Другой распространенной единицей энергии, часто используемой для получения тепла, является калория (кал), определяемая как энергия, необходимая для изменения температуры 1,00 г воды, в частности, между и, поскольку существует небольшая температурная зависимость.Также обычно используется килокалория (ккал), которая представляет собой энергию, необходимую для изменения температуры 1,00 кг воды на. Так как масса чаще всего указывается в килограммах, то килокалория удобна. Как ни странно, пищевые калории (иногда называемые «большими калориями», сокращенно Cal) на самом деле являются килокалориями, что нелегко определить по маркировке упаковки.

Механический эквивалент тепла

Также можно изменить температуру вещества, выполняя работу, которая передает энергию в систему или из нее.Это понимание помогло установить, что тепло - это форма энергии. Джеймс Прескотт Джоуль (1818–1889) провел множество экспериментов, чтобы установить механический эквивалент тепла - работа, необходимая для получения тех же эффектов, что и передача тепла . В единицах, используемых для этих двух величин, эквивалентность равна

.

Мы считаем, что это уравнение представляет преобразование между двумя единицами энергии. (Другие числа, которые вы можете увидеть, относятся к калориям, определенным для температурных диапазонов, отличных от до.)

(рисунок) показывает одну из самых известных экспериментальных установок Джоуля для демонстрации того, что работа и тепло могут производить одни и те же эффекты, и измерения механического эквивалента тепла. Это помогло установить принцип сохранения энергии. Гравитационная потенциальная энергия ( U ) была преобразована в кинетическую энергию ( K ), а затем рандомизирована по вязкости и турбулентности в увеличенную среднюю кинетическую энергию атомов и молекул в системе, что привело к увеличению температуры.Вклад Джоуля в термодинамику был настолько значительным, что в его честь была названа единица энергии в системе СИ.

Эксперимент Джоуля установил эквивалентность тепла и работы. По мере того, как массы спускались, они заставляли весла работать на воде. Результатом стало повышение температуры, измеренное термометром. Джоуль обнаружил, что он пропорционален W и, таким образом, определил механический эквивалент тепла.

Увеличение внутренней энергии за счет теплопередачи дает тот же результат, что и увеличение ее за счет выполнения работы.Следовательно, хотя система имеет четко определенную внутреннюю энергию, мы не можем сказать, что она имеет определенное «теплосодержание» или «рабочее содержание». Четко определенная величина, которая зависит только от текущего состояния системы, а не от истории этой системы, называется переменной состояния . Температура и внутренняя энергия являются переменными состояния. Подводя итог этому абзацу, тепло и работа не являются переменными состояния .

Между прочим, увеличение внутренней энергии системы не обязательно увеличивает ее температуру.Как мы увидим в следующем разделе, температура не меняется, когда вещество переходит из одной фазы в другую. Примером может служить таяние льда, которое может быть достигнуто путем добавления тепла или выполнения работы трения, например, когда кубик льда трется о шероховатую поверхность.

Изменение температуры и теплоемкость

Мы отметили, что теплопередача часто вызывает изменение температуры. Эксперименты показывают, что без изменения фазы и без работы системы или с ее помощью переданное тепло, как правило, прямо пропорционально изменению температуры и массы системы в хорошем приближении.(Ниже мы покажем, как действовать в ситуациях, когда приближение неверно.) Константа пропорциональности зависит от вещества и его фазы, которая может быть газом, жидкостью или твердым телом. Мы опускаем обсуждение четвертой фазы, плазмы, потому что, хотя это наиболее распространенная фаза во Вселенной, она редка и недолговечна на Земле.

Мы можем понять экспериментальные факты, заметив, что передаваемое тепло - это изменение внутренней энергии, которая является полной энергией молекул.В типичных условиях полная кинетическая энергия молекул составляет постоянную долю внутренней энергии (по причинам и за исключениями, которые мы увидим в следующей главе). Средняя кинетическая энергия молекулы пропорциональна абсолютной температуре. Следовательно, изменение внутренней энергии системы обычно пропорционально изменению температуры и количеству молекул, N . Математически зависимость от вещества в значительной степени обусловлена ​​разной массой атомов и молекул.Мы рассматриваем его теплоемкость с точки зрения его массы, но, как мы увидим в следующей главе, в некоторых случаях теплоемкость на молекулу одинакова для разных веществ. Зависимость от вещества и фазы также является результатом различий в потенциальной энергии, связанной с взаимодействиями между атомами и молекулами.

Значения удельной теплоемкости обычно необходимо измерять, потому что нет простого способа их точно рассчитать. (Рисунок) показывает типичные значения теплоемкости для различных веществ.Из этой таблицы видно, что удельная теплоемкость воды в пять раз больше, чем у стекла и в 10 раз больше, чем у железа, что означает, что для повышения температуры воды на определенное количество требуется в пять раз больше тепла, чем у стекла, и в 10 раз больше. столько, сколько по железу. Фактически, вода имеет одну из самых высоких удельной теплоемкости из всех материалов, что важно для поддержания жизни на Земле.

Удельная теплота газов зависит от того, что поддерживается постоянным во время нагрева - обычно от объема или давления.В таблице первое значение удельной теплоемкости для каждого газа измерено при постоянном объеме, а второе (в скобках) измерено при постоянном давлении. Мы вернемся к этой теме в главе, посвященной кинетической теории газов.

Как правило, удельная теплоемкость также зависит от температуры. Таким образом, точное определение c для вещества должно быть дано в терминах бесконечно малого изменения температуры. Для этого отметим это и заменим на d :

За исключением газов, температурная и объемная зависимость удельной теплоемкости большинства веществ слабая при нормальных температурах.Поэтому мы обычно принимаем удельную теплоемкость постоянными на значениях, указанных в таблице.

(рисунок) иллюстрирует повышение температуры, вызванное работой. (Результат такой же, как если бы такое же количество энергии было добавлено с помощью паяльной лампы, а не механически.)

Расчет повышения температуры в результате работы, проделанной на грузовике с веществом. Тормоза, используемые для контроля скорости на спуске, выполняют работу, преобразуя гравитационную потенциальную энергию в повышенную внутреннюю энергию (более высокую температуру) тормозного материала ((рисунок)).Это преобразование предотвращает преобразование потенциальной гравитационной энергии в кинетическую энергию грузовика. Поскольку масса грузовика намного больше массы тормозного материала, поглощающего энергию, повышение температуры может происходить слишком быстро, чтобы тепло от тормозов передавалось в окружающую среду; Другими словами, тормоза могут перегреться.

Дымящиеся тормоза тормозной тележки - видимое свидетельство механического эквивалента тепла.

Рассчитайте повышение температуры 10 кг тормозного материала со средней удельной теплоемкостью, если материал удерживает 10% энергии от спускающегося грузовика массой 10 000 кг 75.0 м (при вертикальном перемещении) с постоянной скоростью.

Стратегия

Мы вычисляем гравитационную потенциальную энергию ( Mgh ), которую весь грузовик теряет при спуске, приравниваем ее к увеличению внутренней энергии тормозов, а затем находим повышение температуры, возникающее только в тормозном материале.

Решение Сначала мы вычисляем изменение гравитационной потенциальной энергии при спуске грузовика:

Поскольку кинетическая энергия грузовика не изменяется, закон сохранения энергии говорит нам, что потерянная потенциальная энергия рассеивается, и мы предполагаем, что 10% ее передается внутренней энергии тормозов, так что возьмите.Затем мы рассчитываем изменение температуры от переданного тепла, используя

, где м - масса тормозного материала. Вставьте указанные значения, чтобы найти

Значение Если бы грузовик ехал какое-то время, то непосредственно перед спуском температура тормозов, вероятно, была бы выше температуры окружающей среды. Повышение температуры при спуске, вероятно, приведет к очень сильному повышению температуры тормозного материала, поэтому этот метод непрактичен.Вместо этого грузовик использовал бы технику торможения двигателем. Другая идея лежит в основе новейшей технологии гибридных и электрических автомобилей, в которой механическая энергия (кинетическая и гравитационная потенциальная энергия) преобразуется тормозами в электрическую энергию в аккумуляторе. Этот процесс называется рекуперативным торможением.

В задачах общего типа объекты с разными температурами контактируют друг с другом, но изолированы от всего остального, и им позволяют прийти в равновесие.Контейнер, который предотвращает передачу тепла внутрь или наружу, называется калориметром, а использование калориметра для измерения (обычно теплоемкости или удельной теплоемкости) называется калориметрией.

Мы будем использовать термин «проблема калориметрии» для обозначения любой проблемы, в которой рассматриваемые объекты термически изолированы от своего окружения. Важная идея при решении задач калориметрии заключается в том, что во время теплообмена между объектами, изолированными от их окружения, тепло, полученное более холодным объектом, должно равняться теплу, теряемому более горячим объектом, из-за сохранения энергии:

Мы выражаем эту идею, записывая, что сумма тепла равна нулю, потому что полученное тепло обычно считается положительным; тепло потеряно, отрицательное.

Расчет конечной температуры в калориметрии. Предположим, вы наливаете 0,250 кг воды (около чашки) в алюминиевую кастрюлю весом 0,500 кг, снятую с плиты, с температурой 0 ° C. Предположим, что теплопередача не происходит ни к чему другому: кастрюлю кладут на изолирующую подкладку, и не учитывают теплопередачу воздуху за короткое время, необходимое для достижения равновесия. Таким образом, это проблема калориметрии, даже если не указан изолирующий контейнер. Также предположим, что выкипает незначительное количество воды.Какова температура, при которой вода и поддон достигают теплового равновесия?

Стратегия Изначально кастрюля и вода не находятся в тепловом равновесии: кастрюля имеет более высокую температуру, чем вода. Теплопередача восстанавливает тепловое равновесие при соприкосновении воды и поддона; он останавливается, когда достигается тепловое равновесие между поддоном и водой. Тепло, теряемое сковородой, равно теплу, полученному водой - это основной принцип калориметрии.

Решение

  1. Используйте уравнение теплопередачи, чтобы выразить тепло, теряемое алюминиевой сковородой, через массу сковороды, удельную теплоемкость алюминия, начальную температуру сковороды и конечную температуру:
  2. Выразите тепло, полученное водой, через массу воды, удельную теплоемкость воды, начальную температуру воды и конечную температуру:
  3. Обратите внимание, что и и что, как указано выше, они должны быть в сумме равными нулю:
  4. Поместите все термины с левой стороны, а все остальные термины с правой стороны.Решение для


    и введите числовые значения:

Значение Почему конечная температура намного ближе к, чем к? Причина в том, что вода имеет большую удельную теплоемкость, чем большинство обычных веществ, и, следовательно, претерпевает меньшее изменение температуры при данной теплопередаче. Большой водоем, например озеро, требует большого количества тепла для значительного повышения температуры. Это объясняет, почему температура в озере остается относительно постоянной в течение дня, даже когда изменение температуры воздуха велико.Однако температура воды действительно меняется в течение длительного времени (например, с лета на зиму).

Проверьте свое понимание Если для повышения температуры породы необходимо 25 кДж, от какого количества тепла необходимо нагреть камень?

В хорошем приближении теплопередача зависит только от разницы температур. Поскольку разница температур в обоих случаях одинакова, во втором случае необходимы те же 25 кДж. (Как мы увидим в следующем разделе, ответ был бы другим, если бы объект был сделан из некоторого вещества, которое меняет фазу где-то между и.)

Температурно-зависимая теплоемкость При низких температурах удельная теплоемкость твердых тел обычно пропорциональна. Первое понимание этого поведения было связано с голландским физиком Питером Дебаем, который в 1912 году рассмотрел атомные колебания с помощью квантовой теории, которую Макс Планк недавно использовал для излучения. Например, хорошее приближение для удельной теплоемкости соли NaCl: Константа 321 K называется температурой Дебая NaCl, и формула хорошо работает, когда Используя эту формулу, сколько тепла требуется для повышения температуры 24.0 г NaCl от 5 К до 15 К?

Решение Поскольку теплоемкость зависит от температуры, нам нужно использовать уравнение

Мы решаем это уравнение для Q путем интегрирования обеих частей:

Затем подставляем данные значения и вычисляем интеграл:

Значение Если бы мы использовали уравнение и удельную теплоемкость соли при комнатной температуре, мы получили бы совсем другое значение.

Понимание расчета Нехера-МакГрата и возможности Кондукторы

Теплопередача

Ключ к пониманию допустимой нагрузки - это понимание теплопередачи.В определение допустимой нагрузки дается в Национальном электротехническом кодексе (NEC) как " ток в амперах, который проводник может нести непрерывно в условиях использовать, не превышая его температурный рейтинг ». Чтобы лучше понять допустимую нагрузку нам необходимо изучить, как передается тепло и как тепловые цепи в относительно проводника с током.

Когда ток проходит по проводнику, он должен проходить через электрическое сопротивление проводника. Когда это происходит, выделяется тепло.Одна единица тепла, ватт, может быть вычислена как квадрат, умноженный на R, где R равно электрическое сопротивление проводника в омах и I равно току в амперы. Тепло, выделяемое в проводнике, проходит через несколько тепловых барьеры конвекцией, проводимостью и излучением и рассеиваются в воздухе. Возможными тепловыми барьерами являются изоляция жилы, воздух внутри воздуховода, стенку воздуховода, грунт вокруг подземного воздуховода и любые дополнительные нанесена теплоизоляция типа полиуретана.

Передача тепла подчиняется фундаментальному закону физики, а теплоотдача всегда течет от более теплого объекта к более холодному, как поток тепла изнутри дома через стены наружу в холодный день. В скорость теплопередачи зависит от нескольких переменных и может быть описана как тепловое уравнение, которое очень похоже на закон Ома (E = IxR), заменяя тепло для тока и термического сопротивления для электрического сопротивления. В жару уравнение переноса скорость теплопередачи напрямую зависит от разница в температуре между проводником, называемым ТС, и окружающей средой температура называется ТА.В уравнении теплопередачи TC-TA = (IxIxR) x RCA, где I - ток в амперах, R - электрическое сопротивление в омах, RCA - термическое сопротивление. сопротивление в градусах Цельсия-см / ватт обычно называют термо-ом-футами. TC это максимально допустимая рабочая температура в градусах Цельсия дирижер. TA - температура окружающего воздуха или почвы для подземных инсталляции. Решение для I:

Допустим, что тепло, в данном случае IxIxR, будет представлено W и термическим сопротивление, RCA, через R с линией над ним, мы можем нарисовать тепловую цепь, которая похожа на электрическую схему.


Уравнение Нехера-МакГрата

Основанный человеком по имени Фурье в 1850-х годах, уравнение № 1 иногда называют уравнением теплопередачи Фурье. Уравнение в разделе 310-15 (b) NEC, называемого уравнением Неера-МакГрата, является более сложным версия уравнения теплопередачи Фурье. Уравнение Неера-МакГрата было обнаружен двумя инженерами-кабелями в 1957 году. В уравнении Неера-МакГрата (Нью-Мексико), Delta TD - это термин, добавляемый к температуре окружающей среды TA, чтобы компенсировать тепло, выделяемое в оболочке и изоляции для более высоких напряжений.Delta TD - это называется повышением температуры диэлектрических потерь и несущественна для напряжений ниже 2000. Другой член в уравнении NM, (1 + YC), является множителем, используемым для преобразовать сопротивление постоянного тока (RDC) в сопротивление переменного тока или сопротивление. Для проволоки размером меньше №2 этот термин становится несущественным. Конечно, мы должны помнить, что уравнение НМ было разработано с использованием стандартная частота сети 60 Гц и синусоидальные формы волны для тока и Напряжение.

Для расчета различных термических сопротивления изоляции проводника, воздушное пространство между проводником и внутренняя часть трубы, труба или стенка воздуховода, а также тепловое сопротивление вне трубопровода.Как и электрические резисторы, термическое сопротивление последовательно складываются, и общая сумма равна RCA.

Температура окружающей среды, TA, варьируется, но обычно составляет 30-40 градусов по Цельсию. используется для наземных установок. Для подземных установок ТА есть повсеместно 20 градусов по Цельсию. Инженеры-строители, работающие в штате Министерство транспорта Аляски заявляет, что фактическая измеренная температура 30 дюймов под поверхностью составляет 19,3 градуса по Цельсию возле Фэрбенкса, Аляска. Это, конечно, в летние месяцы.Температура жилы, TC, для большинства строительных проводов на 600 вольт - 60, 75 или 90 градусов по Цельсию. В максимальная температура изоляции проводов определяется методом старения. и испытания на удлинение в климатических камерах.

В расчетах ЯМ есть много переменных в 30-40 уравнениях. используется для учета количества проводников, количества и размера прилегающих каналов, количество и размер прилегающих рядов каналов, коэффициент поверхности излучательная способность, количество кабелей, осевое расстояние между кабелями, постороннее тепло источники и скорость ветра.Все эти и другие факторы влияют на расчет. емкости. Анализ расчета ЯМ раскрывает много деталей о допустимая нагрузка: например, допустимая нагрузка на проводники в ярком и блестящем кабелепроводе. на открытом воздухе выше допустимой нагрузки в тусклом и темном трубопроводе из-за коэффициент поверхностной излучательной способности и его влияние на тепловое излучение. Также выявлен один из наиболее критикуемых недостатков расчета ЯМ: Расчет основан на одном линейном футе проводника, который может быть несколько сотен футов в длину, где условия резко меняются вдоль Вся длина.

В Национальном электротехническом кодексе есть таблицы допустимой нагрузки, которые достаточно для большинства установок. Однако таблицы в NEC очень грубые приближения и, следовательно, включают значительный запас прочности. Там являются случаями, когда применение таблиц допустимой нагрузки, включая безопасность маржа недостаточна, требуя от инженеров, монтажников и инспекторов выполнять фактические расчеты ЯМ с помощью одного из нескольких программных пакетов имеется в наличии. Например, в NEC нет требований, касающихся проблема чрезмерной теплоизоляции вокруг кабелей и трубопроводов.Что происходит, если вокруг трубы несколько дюймов пенополиуретана? В NEC нет таблиц снижения номинальных характеристик для такого рода ситуаций. Тем не менее, добавление излишней теплоизоляции повлияет на допустимую нагрузку проводник, особенно пенополиуретан, имеющий втрое большую изоляцию стоимость стеклопластика. Чтобы решить эту проблему, мы должны помнить, что NM уравнение представляет собой уравнение радиального теплопереноса, и что расчет NM выполняется на одной типичной опоре установки, которая может быть несколько сотен ноги в длину.Радиальная теплопередача означает, что тепло течет наружу под углом девяносто градусов. длине проводника в отличие от осевой теплопередачи, когда тепло течет по длине проводника. В реальном мире есть осевые и радиальная теплопередача. Но уравнение NM и NEC предполагают, что проводник и окружающие тепловые барьеры бесконечно длинные и однородные там, где нет осевых имеет место теплопередача. Тем не менее, в NEC есть некоторые льготы для осевой теплообмен. Например, более трех токоведущие проводники в ниппеле, если ниппель не более 24 дюймов долго.Кроме того, не требуется снижение номинальных характеристик связанных кабелей, если пучки не длиннее 24 дюймов. Также существует правило десяти процентов, приведенное в разделе 310-15 (с). Это ситуации, когда имеется достаточная осевая теплопередача к не допускать перегрева проводов. Также было бы разумно предположить что там, где есть чрезмерная теплоизоляция длиной не более 24 дюймов, допустимая нагрузка на соответствующие проводники не будет нарушена из-за осевого теплопередача.

11.2 Тепло, удельная теплоемкость и теплопередача - физика

Теплообмен, удельная теплоемкость и теплоемкость

В предыдущем разделе мы узнали, что температура пропорциональна средней кинетической энергии атомов и молекул в веществе, и что средняя внутренняя кинетическая энергия вещества тем выше, чем выше температура вещества.

Если два объекта с разной температурой соприкасаются друг с другом, энергия передается от более горячего объекта (то есть объекта с более высокой температурой) к более холодному (с более низкой температурой) объекту, пока оба объекта не будут иметь одинаковую температуру. .При равенстве температур нетто-передачи тепла, поскольку количество тепла, передаваемого от одного объекта к другому, равно количеству возвращенного тепла. Одним из основных эффектов теплопередачи является изменение температуры: нагревание увеличивает температуру, а охлаждение снижает ее. Эксперименты показывают, что тепло, передаваемое веществу или от него, зависит от трех факторов: изменения температуры вещества, массы вещества и определенных физических свойств, связанных с фазой вещества.

Уравнение теплопередачи Q равно

Q = mcΔT, Q = mcΔT,

11,7

, где м - масса вещества, а Δ T - изменение его температуры в единицах Цельсия или Кельвина. Символ c обозначает удельную теплоемкость и зависит от материала и фазы. Удельная теплоемкость - это количество тепла, необходимое для изменения температуры 1,00 кг массы на 1,00 ºC. Удельная теплоемкость c - это свойство вещества; его единица СИ - Дж / (кг К) или Дж / (кг ° C ° C).Изменение температуры (ΔTΔT) одинаково в кельвинах и градусах Цельсия (но не в градусах Фаренгейта). Удельная теплоемкость тесно связана с понятием теплоемкости. Теплоемкость - это количество тепла, необходимое для изменения температуры вещества на 1,00 ° C ° C. В форме уравнения теплоемкость C равна C = mcC = mc, где m - масса, а c - удельная теплоемкость. Обратите внимание, что теплоемкость такая же, как и удельная теплоемкость, но без какой-либо зависимости от массы.Следовательно, два объекта, состоящие из одного и того же материала, но с разной массой, будут иметь разную теплоемкость. Это связано с тем, что теплоемкость - это свойство объекта, а удельная теплоемкость - это свойство любого объекта , изготовленного из того же материала.

Значения удельной теплоемкости необходимо искать в таблицах, потому что нет простого способа их вычислить. В таблице 11.2 приведены значения удельной теплоемкости для некоторых веществ в качестве справочной информации. Из этой таблицы видно, что удельная теплоемкость воды в пять раз больше, чем у стекла, а это означает, что для повышения температуры 1 кг воды требуется в пять раз больше тепла, чем для повышения температуры 1 кг стекла тем же самым способом. количество градусов.

Поддержка учителей

Поддержка учителей

[BL] [OL] [AL] Объясните, что эта формула работает только в том случае, если фаза вещества не меняется. Передача тепловой энергии, тепла и фазовый переход будут рассмотрены позже в этой главе.

Предупреждение о заблуждении

Единицы измерения удельной теплоемкости - Дж / (кг ° C⋅ ° C) и Дж / (кг K). Однако градусы Цельсия и Кельвина не всегда взаимозаменяемы. В формуле для удельной теплоемкости используется разница в температуре, а не абсолютная температура.Это причина того, что градусы Цельсия могут использоваться вместо Кельвина.

Вещества Удельная теплоемкость ( c )
Твердые вещества Дж / (кг ⋅ ° C⋅ ° C)
Алюминий 900
Асбест 800
Бетон, гранит (средний) 840
Медь 387
Стекло 840
Золото 129
Человеческое тело (среднее) 3500
Лед (средний) 2090
Чугун, сталь 452
Свинец 128
Серебро 235
Дерево 1700
Жидкости
Бензол 1740
Этанол 2450
Глицерин 2410
Меркурий 139
Вода 4186
Газы (при постоянном давлении 1 атм)
Воздух (сухой) 1015
Аммиак 2190
Двуокись углерода 833
Азот 1040
Кислород 913
Пар 2020

Таблица 11.2 Удельная теплоемкость различных веществ.

Snap Lab

Изменение температуры земли и воды

Что нагревается быстрее, земля или вода? Вы ответите на этот вопрос, проведя измерения для изучения различий в удельной теплоемкости.

  • Открытое пламя. Соберите все распущенные волосы и одежду, прежде чем зажечь открытое пламя. Следуйте всем инструкциям своего учителя о том, как зажечь пламя. Никогда не оставляйте открытое пламя без присмотра. Знайте расположение противопожарного оборудования в лаборатории.
  • Песок или грунт
  • Вода
  • Духовка или тепловая лампа
  • Две маленькие баночки
  • Два термометра

Инструкции

Процедура

  1. Поместите равные массы сухого песка (или почвы) и воды одинаковой температуры в две небольшие банки. (Средняя плотность почвы или песка примерно в 1,6 раза больше плотности воды, поэтому вы можете получить равные массы, используя на 50 процентов больше воды по объему.)
  2. Нагрейте оба вещества (с помощью духовки или нагревательной лампы) в течение одинакового времени.
  3. Запишите конечные температуры двух масс.
  4. Теперь доведите обе банки до одинаковой температуры, нагревая их в течение более длительного периода времени.
  5. Снимите банки с источника тепла и измеряйте их температуру каждые 5 минут в течение примерно 30 минут.

Проверка захвата

Потребовалось больше времени, чтобы нагреть воду или песок / почву до той же температуры? Какой образец остыл дольше? Что этот эксперимент говорит нам о том, как удельная теплоемкость воды по сравнению с удельной теплотой земли?

  1. Песок / почва нагревается и остывает дольше.Это говорит нам о том, что удельная теплоемкость земли больше, чем у воды.
  2. Песок / почва нагревается и остывает дольше. Это говорит нам о том, что удельная теплоемкость воды больше, чем у земли.
  3. Вода нагревается и остывает дольше. Это говорит нам о том, что удельная теплоемкость земли больше, чем у воды.
  4. Вода нагревается и остывает дольше. Это говорит нам о том, что удельная теплоемкость воды больше, чем у земли.

Проводимость, конвекция и излучение

Всякий раз, когда есть разница температур, происходит теплопередача. Передача тепла может происходить быстро, например, через сковороду, или медленно, например, через стенки изолированного холодильника.

Существует три различных метода теплопередачи: теплопроводность, конвекция и излучение. Иногда все три могут происходить одновременно. См. Рисунок 11.3.

Рис. 11.3 В камине передача тепла происходит всеми тремя способами: теплопроводностью, конвекцией и излучением.Излучение отвечает за большую часть тепла, передаваемого в комнату. Передача тепла также происходит через теплопроводность в комнату, но гораздо медленнее. Теплообмен за счет конвекции также происходит через холодный воздух, поступающий в комнату вокруг окон, и горячий воздух, покидающий комнату, поднимаясь вверх по дымоходу.

Проводимость - это передача тепла при прямом физическом контакте. Тепло, передаваемое между электрической горелкой плиты и дном сковороды, передается за счет теплопроводности. Иногда мы пытаемся контролировать теплопроводность, чтобы чувствовать себя более комфортно.Поскольку скорость теплопередачи у разных материалов разная, мы выбираем такие ткани, как толстый шерстяной свитер, которые зимой замедляют отвод тепла от нашего тела.

Когда вы идете босиком по ковру в гостиной, ваши ноги чувствуют себя относительно комфортно… пока вы не ступите на кафельный пол кухни. Поскольку ковер и кафельный пол имеют одинаковую температуру, почему один из них холоднее другого? Это объясняется разной скоростью теплопередачи: материал плитки отводит тепло от вашей кожи с большей скоростью, чем ковровое покрытие, что делает его на более холодным.

Поддержка учителей

Поддержка учителей

[BL] [OL] [AL] Спросите учащихся, какая сейчас температура в классе. Спросите их, все ли предметы в комнате имеют одинаковую температуру. Как только это будет установлено, попросите их положить руку на стол или на металлический предмет. Стало холоднее? Почему? Если их стол сделан из ламината Formica, то рука будет прохладно, потому что ламинат является хорошим проводником тепла и отводит тепло от руки, создавая ощущение «холода» из-за тепла, покидающего тело.

Некоторые материалы просто проводят тепловую энергию быстрее, чем другие. В общем, металлы (например, медь, алюминий, золото и серебро) являются хорошими проводниками тепла, тогда как такие материалы, как дерево, пластик и резина, плохо проводят тепло.

На рис. 11.4 показаны частицы (атомы или молекулы) в двух телах при разных температурах. (Средняя) кинетическая энергия частицы в горячем теле выше, чем в более холодном теле. Если две частицы сталкиваются, энергия передается от частицы с большей кинетической энергией к частице с меньшей кинетической энергией.Когда два тела находятся в контакте, происходит много столкновений частиц, что приводит к чистому потоку тепла от тела с более высокой температурой к телу с более низкой температурой. Тепловой поток зависит от разности температур ΔT = Thot-TcoldΔT = Thot-Tcold. Таким образом, вы получите более сильный ожог от кипятка, чем от горячей воды из-под крана.

Рис. 11.4. Частицы в двух телах при разных температурах имеют разные средние кинетические энергии. Столкновения, происходящие на контактной поверхности, имеют тенденцию передавать энергию из высокотемпературных областей в низкотемпературные области.На этой иллюстрации частица в области более низких температур (правая сторона) имеет низкую кинетическую энергию перед столкновением, но ее кинетическая энергия увеличивается после столкновения с контактной поверхностью. Напротив, частица в области более высоких температур (слева) имеет большую кинетическую энергию до столкновения, но ее энергия уменьшается после столкновения с контактной поверхностью.

Конвекция - это передача тепла движением жидкости. Такой тип теплопередачи происходит, например, в котле, кипящем на плите, или во время грозы, когда горячий воздух поднимается к основанию облаков.

Советы для успеха

В обиходе термин жидкость обычно означает жидкость. Например, когда вы заболели и врач говорит вам «выпить жидкости», это означает только пить больше напитков, а не вдыхать больше воздуха. Однако в физике жидкость означает жидкость или газ . Жидкости движутся иначе, чем твердые тела, и даже имеют свой собственный раздел физики, известный как гидродинамика , который изучает их движение.

При повышении температуры жидкости они расширяются и становятся менее плотными.Например, на рис. 11.4 может быть изображена стенка воздушного шара с газами внутри воздушного шара с другой температурой, чем снаружи в окружающей среде. Более горячие и, следовательно, быстро движущиеся частицы газа внутри воздушного шара ударяются о поверхность с большей силой, чем более холодный воздух снаружи, вызывая расширение воздушного шара. Это уменьшение плотности по отношению к окружающей среде создает плавучесть (тенденцию к повышению). Конвекция обусловлена ​​плавучестью - горячий воздух поднимается вверх, потому что он менее плотен, чем окружающий воздух.

Иногда мы контролируем температуру своего дома или самих себя, контролируя движение воздуха. Герметизация дверей герметичным уплотнением защищает от холодного ветра зимой. Дом на рис. 11.5 и горшок с водой на плите на рис. 11.6 являются примерами конвекции и плавучести, созданными человеком. Океанские течения и крупномасштабная атмосферная циркуляция переносят энергию из одной части земного шара в другую и являются примерами естественной конвекции.

Рисунок 11.5 Воздух, нагретый так называемой гравитационной печью, расширяется и поднимается вверх, образуя конвективную петлю, которая передает энергию другим частям комнаты. По мере того, как воздух охлаждается у потолка и внешних стен, он сжимается, в конечном итоге становясь более плотным, чем воздух в помещении, и опускается на пол. Правильно спроектированная система отопления, подобная этой, в которой используется естественная конвекция, может быть достаточно эффективной для равномерного обогрева дома.

Рис. 11.6 Конвекция играет важную роль в теплопередаче внутри этого сосуда с водой.Попав внутрь жидкости, теплопередача к другим частям кастрюли происходит в основном за счет конвекции. Более горячая вода расширяется, уменьшается по плотности и поднимается, передавая тепло другим областям воды, в то время как более холодная вода опускается на дно. Этот процесс повторяется до тех пор, пока в кастрюле есть вода.

Излучение - это форма передачи тепла, которая происходит при испускании или поглощении электромагнитного излучения. Электромагнитное излучение включает радиоволны, микроволны, инфракрасное излучение, видимый свет, ультрафиолетовое излучение, рентгеновские лучи и гамма-лучи, все из которых имеют разные длины волн и количество энергии (более короткие волны имеют более высокую частоту и больше энергии).

Поддержка учителей

Поддержка учителей

[BL] [OL] Электромагнитные волны также часто называют электромагнитными волнами. Мы по-разному воспринимаем электромагнитные волны разной частоты. Так же, как мы можем видеть одни частоты как видимый свет, мы воспринимаем некоторые другие как тепло.

Вы можете почувствовать теплоотдачу от огня и солнца. Точно так же вы иногда можете сказать, что духовка горячая, не касаясь ее дверцы и не заглядывая внутрь - она ​​может просто согреть вас, когда вы пройдете мимо.Другой пример - тепловое излучение человеческого тела; люди постоянно излучают инфракрасное излучение, которое не видно человеческому глазу, но ощущается как тепло.

Излучение - единственный метод передачи тепла, при котором среда не требуется, а это означает, что тепло не должно вступать в прямой контакт с какими-либо предметами или переноситься ими. Пространство между Землей и Солнцем в основном пусто, без какой-либо возможности теплопередачи за счет конвекции или теплопроводности. Вместо этого тепло передается за счет излучения, и Земля нагревается, поскольку она поглощает электромагнитное излучение, испускаемое Солнцем.

Рис. 11.7 Большая часть тепла от этого пожара передается наблюдателям через инфракрасное излучение. Видимый свет передает относительно небольшую тепловую энергию. Поскольку кожа очень чувствительна к инфракрасному излучению, вы можете почувствовать присутствие огня, даже не глядя на него. (Дэниел X. О’Нил)

Все объекты поглощают и излучают электромагнитное излучение (см. Рисунок 11.7). Скорость передачи тепла излучением в основном зависит от цвета объекта. Черный - наиболее эффективный поглотитель и радиатор, а белый - наименее эффективный.Например, люди, живущие в жарком климате, обычно избегают ношения черной одежды. Точно так же черный асфальт на стоянке будет горячее, чем прилегающие участки травы в летний день, потому что черный поглощает лучше, чем зеленый. Верно и обратное - черный цвет излучает лучше, чем зеленый. Ясной летней ночью черный асфальт будет холоднее, чем зеленый участок травы, потому что черный излучает энергию быстрее, чем зеленый. Напротив, белый цвет - плохой поглотитель и плохой радиатор. Белый объект, как зеркало, отражает почти все излучение.

Поддержка учителя

Поддержка учителя

Попросите учащихся привести примеры теплопроводности, конвекции и излучения.

Виртуальная физика

Формы и изменения энергии

В этой анимации вы исследуете теплопередачу с различными материалами. Поэкспериментируйте с нагревом и охлаждением железа, кирпича и воды. Для этого перетащите объект на пьедестал и затем удерживайте рычаг в положении «Нагреть» или «Охлаждать». Перетащите термометр рядом с каждым объектом, чтобы измерить его температуру - вы можете в реальном времени наблюдать за тем, как быстро он нагревается или охлаждается.

Теперь попробуем передать тепло между объектами. Нагрейте кирпич и поместите его в прохладную воду. Теперь снова нагрейте кирпич, но затем поместите его поверх утюга. Что ты заметил?

Выбор опции быстрой перемотки вперед позволяет ускорить передачу тепла и сэкономить время.

Проверка захвата

Сравните, насколько быстро различные материалы нагреваются или охлаждаются. Основываясь на этих результатах, какой материал, по вашему мнению, имеет наибольшую удельную теплоемкость? Почему? Какая из них имеет наименьшую удельную теплоемкость? Можете ли вы представить себе реальную ситуацию, в которой вы хотели бы использовать объект с большой удельной теплоемкостью?

  1. Вода занимает больше всего времени, а железу нужно меньше времени, чтобы нагреться и остыть.Для теплоизоляции желательны объекты с большей удельной теплоемкостью. Например, шерстяная одежда с большой удельной теплоемкостью предотвратит потерю тепла телом.
  2. Вода занимает меньше всего времени, а железу нужно больше времени, чтобы нагреться и остыть. Для теплоизоляции желательны объекты с большей удельной теплоемкостью. Например, шерстяная одежда с большой удельной теплоемкостью предотвратит потерю тепла телом.
  3. Кирпич занимает меньше всего времени, а железу нужно больше времени, чтобы нагреться и остыть.Для теплоизоляции желательны объекты с большей удельной теплоемкостью. Например, шерстяная одежда с большой удельной теплоемкостью предотвратит потерю тепла телом.
  4. Вода занимает меньше всего времени, а кирпичу нужно больше времени, чтобы нагреться и остыть. Для теплоизоляции желательны объекты с большей удельной теплоемкостью. Например, шерстяная одежда с большой удельной теплоемкостью предотвратит потерю тепла телом.
Поддержка учителей
Поддержка учителей

Попросите учащихся рассмотреть различия в результатах интерактивных упражнений при использовании разных материалов.Например, спросите их, было бы изменение температуры больше или меньше, если бы кирпич был заменен железным блоком той же массы, что и кирпич.

alexxlab

Добавить комментарий

Ваш адрес email не будет опубликован. Обязательные поля помечены *